Тормозной путь в физике формула: Как рассчитать тормозной путь — О’Пять пО физике!

Содержание

Конспект урока по физике «Тормозной путь» (9 класс)

Тема: «Тормозной путь. Остановочный путь. Безопасность дорожного движения»

Цели урока:

  • Опираясь на знание физических законов, выработать осознанную необходимость соблюдения Правил дорожного движения. Познакомить учащихся с понятиями тормозной и остановочный путь.

  • Воспитывать законопослушность, ответственность за свою жизнь и жизнь людей, живущих рядом. Повторить основные знания по безопасному переходу улицы.

  • Развивать творческие способности, коммуникабельность

Ход урока:

  1. Орг. момент

Учитель: Часто на дорогах можно видеть перебегающего пешехода впереди идущего транспорта. По — видимому, пешеход не знает, что достаточно 30 секунд постоять на обочине, перекрестке перед красным сигналом светофора и для него загорится зеленый свет. Еще он не знает элементарных сведений о тормозном пути автомашин и не соблюдает Правила пешехода.

Сегодняшний урок посвящен взаимосвязи физики, математики и безопасности дорожного движения, мы познакомимся с понятиями тормозной путь и остановочный путь, повторим правила дорожного движения. Поэтому тема урока звучит так: «Тормозной путь. Остановочный путь. Безопасность дорожного движения» Слайд №1

Эпиграфом к нашему уроку я выбрала слова великого русского писателя А.П.Чехова «Солнце не всходит два раза в день, а жизнь не дается дважды…» Слайд №2

Проблема безопасности движения сложна и многогранна, вы каждый день идете или едете в школу, т.е. являетесь участниками дорожного движения.

  1. Актуализация знаний

Давайте посмотрим на фотографию, и наверное, тогда станет ясно, почему мы сегодня говорим о безопасности дорожного движения. Слайд № 3

Слайд № 4 Ежегодно в мире в результате ДТП погибают и получают ранения более 50 млн. человек. Всемирная организация здравоохранения свидетельствует, что на долю ДТП более 30% смертельных исходов от всех несчастных случаев. В ХХ веке автомобиль стал причиной смерти около 30 млн. человек. В России потери, связанные с ДТП в несколько раз превышают ущерб от железнодорожных катастроф, пожаров и других видов несчастных случаев. Масштаб ДТП угрожает национальной безопасности.

Слайд № 5 Мы каждое утро выходим на улицу и становимся пешеходами. Анализ данных о количестве ДТП свидетельствует о том, что очень часто жертвами становятся пешеходы, в том числе и дети.

Исследования пешеходного движения показывают, что в зависимости от возраста и пола изменяется и скорость движения пешеходов:

Слайд № 6 Скорость передвижения пешеходов.

Таблица 1. Возраст и пол Скорость движения пешеходов

Дети 6-10 лет 1,11 м/с = 1,11X3600 : 1000 км/ч = 3,996 км/ч

Подростки 11-16 лет 1,59 м/с = 1.59X3600 : 1000 км/ч = 5,724 км/ч

Мужчины до 55 лет 1,62 м/с = 1.62 X3600: 1000 км/ч = 5,832 км/ч

Мужчины свыше 55лет 1,5 м/с = 1.5 X3600 : 1000 км/ч = 5,4 км/ч

Женщины до 55 лет 1,35 м/с = 1.35X3600 : 1000 км/ч = 4,86 км/ч

Женщины после 55 лет 1,29 м/с = 1.29X3600 : 1000 км/ч = 4,644км/ч

Слайд № 7 Пусть вам сообщили, что скорость мужчины 6 км/ч. Что означает это число?

Дорога — объект повышенной опасности. Помогают избежать опасных ситуаций на дорогах, конечно, дорожные знаки. Что означают эти знаки?

Слайд № 8 Вы ехали по городу и увидели знак ограничения скорости 40 км/ч, а в городе дует штормовой ветер, то есть где-то 25 – 30 м/с. Нарушает ли ветер правила дорожного движения? Что нужно нам сделать, чтобы сравнить эти две величины?

Выразим скорость движения ветра в других единицах (км/ч).

Давайте вспомним, что нужно для этого сделать: 30м/с = = 108 км/ч

Да. Нарушает.

Задача.

В начале участка шоссе стоит дорожный знак «30 км/ч». Нарушил ли правила движения водитель автомобиля, равномерно прошедшего участок дороги длиной 1,8 км за 4 мин? (Не нарушил, т. к. скорость автомобиля была 27 км/ч).

Почему дети часто становятся жертвами ДТП?

Слайд № 9 1. При переходе дороги для того, чтобы иметь общее представление об окружающем пространстве, нужно осмотреться вокруг. Для того чтобы повернуть голову, ребёнку понадобится 4 секунды, а взрослому человеку всего лишь одну секунду.

Поэтому, искажённо оценив дорожную ситуацию, дети считают, что успеют перейти дорогу и попадают в ДТП.

Слайд № 10 2. Дети с искажением воспринимают звуки на дороге.

Подростки часто ходят по улицам города с наушниками, в которых звучит громкая музыка. И это очень существенно мешает восприятию окружающей действительности.

Слайд № 11 3. У ребёнка искажено восприятие размеров транспортного средства. Подростки до 13-14 лет видят только прямо. У них хорошо развито “тоннельное зрение”, а боковое зрение слабо фиксирует происходящее.

Слайд № 12 Решить задачу. Сколько секунд будет переходить ребёнок дорогу, если её ширина 6 метров и его скорость равна 1,2 м/с?

  1. Изучение нового материала

1. Учитель. Ребята, помните – ни одна машина сразу остановиться не может. Знание того, сколько проедет автомобиль перед тем, как остановиться, какова дистанция безопасности, поможет избежать проблем на дороге.

1) Останавливаясь около заранее намеченного места, водитель выключает двигатель, и автомашина продолжает двигаться только по инерции. Затем водитель тормозит и плавно подводит машину к остановке.

2) Автомашина не может остановиться сразу, как только её затормозили. До полной остановки она проходит ещё некоторое расстояние – тормозной путь.

Слайд 13. Тормозной путь.

Определение 1. Тормозной путь — это расстояние, пройденное автомобилем с момента нажатия на педаль тормоза до полной остановки автомобиля.

Вопросы. От чего зависит тормозной путь автомобиля?

Тормозной путь зависит от многих факторов:

а) от силы сцепления колёс с землёй. Если дорога скользкая, шины стёрты, то сила сцепления колёс с землёй уменьшается и, наоборот, увеличивается, если дорога сухая, а шины новые. В первом случае тормозной путь увеличивается, во втором сокращается.

б) от скорости автомобиля: чем больше скорость, тем он длиннее.

Слайд 14. Изменение тормозного пути.

Водитель едет со скоростью 25 км/ч, затем переходит на скорость 50 км/ч.

Как изменится путь торможения?

Слайд 15 Изменение тормозного пути от увеличения скорости

Казалось бы, что при удвоенной скорости тормозной путь тоже должен увеличиться вдвое. На самом деле он увеличивается в 4 раза. Если скорость увеличить в 3 раза, то тормозной путь увеличивается в 9 раз, а если в 4 раза – то в 16 раз.

Почему так происходит? Чтобы ответить на этот вопрос выведем формулу зависимости пути от скорости.

Слайд 16 — Если тело движется, то какой энергией оно обладает?

Кинетической энергией Ек = (1)

— Если тело обладает энергией и энергия уменьшается, значит тело выполняет работу, равную изменению этой энергии, т.е. А = (2)

— Какие силы действуют на автомобиль, если двигатель выключен?

Fтр = (3)

А = Fтрs = (4)

Приравняем 2 и 4 формулы

=

Задание командам из этой формулы вывести формулу пути.

= =

Слайд 17 РЕШИМ ЗАДАЧУ: Определим тормозной путь при экстренном торможении.

Пешеход пересекает улицу в неположенном месте. Водитель замечает пешехода за 20 м и начинает экстренное торможение. Произойдёт ли аварии, если скорость авто 60 км/ч? Коэффициент трения 0,7.

60 км/ч = 16,7 м/с

Решение: рассчитаем тормозной путь по выведенной формуле:

=

S= 19,9 м.

В данной ситуации всё обошлось, а что было бы, если за 5 минут до этого прошёл дождь? Коэффициент трения = 0,5

S= 27,8 м. Машина собьёт человека.

Как изменяется тормозной путь в зависимости от скорости движения автомашины, показано в таблице.

в) от состояния дороги.

Числа в таблице приведены для сухой асфальтовой дороги.

Слайд 18. Тормозной путь.

Тормозной путь может увеличиться примерно на 30%, если дорога мокрая, и примерно в 3 раза, если дорога покрыта снегом, и в 5 раз, если асфальт покрыт ледяной коркой.

В таблице указан тормозной путь по 1) мокрой дороге, 2) дороге, покрытой ледяной коркой.

Слайд 19. Тормозной путь.

г) от нагрузки и тяжести машины. Более тяжёлая машина (грузовик, автобус) имеет больший тормозной путь, чем, например, маленький “Москвич”.

д) от исправности тормозов, препятствий на пути и других условий.

  1. Закрепление.

Слайд 20. От чего зависит тормозной путь

а ) от силы сцепления колёс с землёй,

б) от скорости автомобиля,

в) от состояния дороги.

г) от нагрузки и тяжести машины

2. Введение понятия остановочный путь автомобиля.

Учитель. Ребята, вы увидели, что автомобиль нельзя остановить мгновенно. Для его остановки требуется определённое время, за которое он проходит некоторое расстояние.

Слайд 21 Остановочный путь.

Определение 2. Все расстояние, пройденное автомобилем с момента обнаружения опасности до полной остановки, называется остановочный путь автомобиля.

Повторим Определение 1. Тормозной путь – это расстояние, пройденное автомобилем с момента нажатия на педаль тормоза до полной остановки автомобиля.

Слайд 22. Составляющие остановочного пути.

Остановочный путь состоит из двух частей: расстояние, пройденное автомобилем за время реакции водителя и тормозного пути.

Путь за время

реакции водителя Тормозной путь

Остановочный путь

Время реакции водителя.

Время реакции водителя колеблется от 0,5 с до 1,2 с. Что влияет на время реакции водителя?

Ответ. На время реакции водителя влияют личные качества: физическое состояние водителя, его возраст, водительский опыт.

Вопрос. Правильно ли, что чем больше скорость автомобиля, тем длиннее остановочный путь?

Ответ. Правильно. Чем больше скорость, тем больше инерция. Тормозной путь удлиняется, а, значит, удлиняется остановочный путь.

Слайд 23.

Задача.

Время реакции водителя на возникшую опасность составляет 0.8 с. Какой путь пройдет за это время автобус, если скорость его была 54 км/ч? (12м).

Слайд 24.

Задача.

Автомобиль движется со скоростью 40 км/ч. На расстоянии 15м у него возникает препятствие. Свернуть некуда. Реакция водителя 0,5 сек. Успеет ли водитель остановить машину? (Дорога сухая.)

Решение.

По таблице находим, что при скорости 40км/ч путь торможения по сухой дороге равен 10,4 м. До препятствия 15 м, значит, у водителя в запасе 15 м – 10,4 м = 4,6 м. Кажется, что автомашину можно успеть остановить. Но здесь мы не учли скорость реакции водителя. Если она составляет 0,5 сек, то при скорости 40км/ч автомашина за это время проедет

40 х 0,5 х 1000 : 3600 м = 5,6 м, а затем ещё 10,4 м до остановки, всего 16 м. А так как до препятствия 15 м, то водитель обязательно на него наедет.

Ответ. Не успеет.

Теперь становится ясно, ребята, какой опасности подвергается пешеход, пытающийся пересечь дорогу, по которой мчатся автомашины.

Слайд 25. Запомни.

  • “Не перебегай улиц и дорог перед близко движущимся транспортом – это очень опасно для жизни”.

  • Ни одно даже самое важное дело не стоит вашей жизни.

Вопрос. Почему нельзя перебегать улицу перед близко идущим транспортом?

Слайд 26. Памятка.

Умей не только видеть, но и слышать улицу.

Обращай внимание на сигналы автомобиля (указатели поворота, заднего хода, тормоза)

Контролируй свои движения: поворот головы для осмотра дороги, остановку для пропуска автомобиля.

  1. Подведение итогов:

– Мы сегодня с вами не только повторили основные физические понятия, такие как скорость, тормозной путь, инерция и трение, но и рассмотрели их практическое применение, повторили правила дорожного  движения и дорожные знаки. Надеюсь, что данные знания помогут вам в жизни.

1. Если увеличить скорость транспорта вдвое, то потребуется вчетверо больший путь до его остановки, т.е. тормозной путь увеличится в 4 раза, а время торможения – в 2 раза.

2. Чем больше масса транспортного средства, тем время торможения и тормозной путь больше, т.е. тем труднее изменить скорость автомобиля и, следовательно, тело более инертно.

3. Длина тормозного пути зависит от погодных условий: на мокрой, скользкой дороге сила сцепления колёс с дорогой уменьшается, а тормозной путь увеличивается.

4. Зависимость тормозного пути от тормозной системы, изношенности шин колёс, освещённости дороги и других факторов.

5. Для остановки транспорта требуется время и пространство: нельзя переходить дорогу перед близко идущим транспортом. Об этом следует помнить во избежание ДТП, как пешеходам, так и автомобилистам, велосипедистам и другим участникам движения.

Наиболее распространенные нарушения:

1) Переход перед близко идущим транспортом;

2) Внезапный выход на проезжую часть из-за стоящего транспорта;

3) Переход дороги вне пешеходного перехода;

4) Нарушение правил езды на велосипеде;

5) Игра на дороге.

Скоро растает снег и многие из вас начнут кататься на велосипедах. Весна радует нас своим приходом. Голубое небо, яркое солнце, таяние снега — всё это поднимает настроение после долгой, суровой зимы. В связи с потеплением на дорогах появится и больше транспорта, что влияет на безопасность дорожного движения. Но, соблюдая установленные правила, мы тем самым оберегаем себя и окружающих от возможных неприятных последствий

(Выставление оценок активным ученикам)

Слайд 26. Домашнее задание:

  1. Составить кроссворд содержащий основные физические понятия и понятия из правил дорожного движения.

  2. Задача. На участке дороги, где установлен такой знак, водитель применил аварийное торможение. Инспектор обнаружил по следу колёс, что тормозной путь равен 12 м. Нарушил ли водитель правила, если коэффициент трения 0,6?

Вот как можно рассчитать тормозной путь: Формула

Как рассчитать расстояние тормозного пути автомобиля.

 

Как быстро автомобиль ускоряется, наверное, знает большинство автовладельцев. Даже если вы не замеряли динамику разгона своей машины, вы наверняка смотрели заводские технические характеристики вашего авто, где обычно автопроизводитель указывает минимально возможное время разгона с 0-100 км/час. Но теперь вопрос: сколько времени нужно, чтобы остановить вашу машину? Вы знаете это? Уверены, что нет. Но, оказывается, рассчитать расстояние тормозного пути можно достаточно легко с помощью простой формулы. Мы расскажем вам, как это делается. 

 

Нет такой вещи во Вселенной или материи, которая может мгновенно остановиться. Также и любой автомобиль, когда вы нажимаете педаль тормоза, не сразу может остановиться. Дело в том, что для того чтобы автомобиль или любой объект в нашем мире остановился, необходимо, чтобы он потерял энергию, которая его движет. В результате у любого автомобиля есть тормозной путь, который он проезжает с момента нажатия педали тормоза до момента полной остановки. Это и есть тормозное расстояние машины.

 

Но на самом деле тормозной путь любого авто зависит не только от его характеристик и тормозной системы, но и от реакции водителя при нажатии педали тормоза. Ведь для того чтобы принять решение о необходимости торможения и нажать педаль тормоза, требуется время, которое хоть и минимально, но достаточно, чтобы машина успела проехать немаленький путь. Особенно это важно при большой скорости движения, где за какие-то доли секунды автомобиль проезжает приличное расстояние. Итак, в итоге, чтобы рассчитать реальную длину тормозного пути, нужно учитывать не только время и расстояние, пройденное автомобилем с момента нажатия водителем педали тормоза до момента остановки машины, но и время, необходимое для принятия решения о торможении. Дело в том, что при принятии решения о торможении мы тратим драгоценные секунды. Вот пример:

 

  • Время отклика: Прежде чем водитель нажмет педаль тормоза, он должен оценить дорожную ситуацию и определить, необходимо ли торможение. Также нужно понять, какое необходимо торможение – полная остановка автомобиля или простое снижение скорости. Обычно, согласно многочисленным исследованиям, большинству водителей для этого требуется около 0,1 секунды. 
  • Время, необходимое для нажатия педали тормоза: После того, как водитель понял, что должен тормозить, необходимо еще примерно 0,8 секунды, для того чтобы переместить ногу с педали газа на педаль тормоза и нажать ее. 

 

Кроме того, даже при нажатии педали тормоза есть еще небольшая потеря времени, связанная с тем, что при нажатии педали тормоза автомобиль, как правило, не начинает резко тормозить. А для того чтобы машина реально начала резко снижать скорость, надо усилить давление на педаль тормоза (пороговое время, необходимое для требуемого тормозного давления в тормозной системе). Также у всех автомобилей разное время отклика на нажатую педаль тормоза. Здесь все, конечно, зависит от конструкции тормозной системы и наличия различной электроники, контролирующей тормоза автомобиля.

 

Смотрите также: Полный привод оказался лучше при торможении, чем привод на два колеса: Видео

 

Вы не поверите, но для того чтобы машина реально начала тормозить после нажатия педали тормоза, необходима еще почти 1 секунда времени. Вы представляете, как это много при движении на большой скорости? За эту лишнюю секунду вы можете проехать очень большой путь.  

 

Что такое формула тормозного пути?

В общем, торможение автомобиля делится на два вида. Например, есть нормальное торможение, а есть экстренное, когда вам нужно резко остановить машину, чтобы избежать аварии.

 

При торможении в повседневной жизни, допустим, если вы хотите остановить автомобиль на светофоре, вы обычно нажимаете педаль тормоза намного плавнее и мягче, чем при необходимости полностью остановить автомобиль на парковке во дворе. В этом случае вы не применяете в машине максимальное тормозное усилие. При таком плавном и мягком торможении, как правило, тормозной путь (тормозное расстояние) увеличивается. Примерное расстояние тормозного пути при нормальном торможении можно рассчитать по следующей простой формуле:

 

(Скорость в км/ч : 10) x (скорость в км/ч : 10) = тормозной путь в метрах

 

При экстренном торможении педаль тормоза, как правило, нажата целиком и с полной силой. Из-за более высокой силы торможения обычно тормозной путь машины сокращается примерно в 2 раза. Поэтому длину тормозного пути можно также вычислить по следующей формуле:

 

(Скорость в км/ч : 10) x (скорость в км/ч : 10) / 2 = тормозной путь в метрах

 

Внимание: Вычисляемый по этим формулам тормозной путь является лишь приблизительным значением и подсказкой для водителей. На самом деле в реальности тормозной путь может быть как меньше, так и больше. Ведь расстояние тормозного пути зависит от навыков и опыта вождения водителя, от технической исправности автомобиля, его конструкции, марки, модели, состояния дорог, состояния протектора резины и многих других факторов, которые напрямую влияют на длину тормозного пути. Но благодаря этим формулам вы примерно сможете высчитать среднюю длину тормозного пути машины при определенной скорости движения. Это позволит вам скорректировать ваш стиль управления автомобилем, а также станет хорошим пособием для водителей-новичков.  

 

Как рассчитать полное время остановки и итоговый тормозной путь?

 

Как мы уже сказали, чтобы рассчитать весь тормозной путь, нужно учитывать потерю времени при принятии водителем решения о торможении (то есть время реакции водителя). Для этого нужно использовать другую формулу, которая обеспечивает более точный приблизительный расчет тормозного расстояния, которое проедет автомобиль в момент принятия решения о необходимости остановки. Вот эта формула:

 

(Скорость в км/ч : 10) x 3 = путь реакции в метрах

 

В итоге, сделав вычисление по вышеуказанным формулам, вы можете вычислить приблизительный итоговый тормозной путь вашего автомобиля при любой скорости движения. Вот пример. Если вы управляете своим автомобилем со скоростью 50 км/ч, то с помощью приведенных формул вычислите следующие значения:

 

  • Тормозной путь при принятии решения о торможении на этой скорости (реакция на дорожную ситуацию + принятие решения о торможении + время, необходимое для перемещения ноги с педали газа на педаль тормоза, а также время отклика тормозной системы на нажатую педаль тормоза) составит где-то (50/10) х 3 = 15 метров. То есть пока вы будете принимать решение о торможении при скорости в 50 км/ч, ваша машина проедет 15 метров. 
  • Тормозной путь при нормальном торможении (с момента нажатия педали тормоза до момента остановки машины) составит около (50/10) х (50/10) = 25 метров. 
  • При экстренном торможении тормозной путь, как мы уже отметили, сокращается примерно в два раза. Соответственно, расчет тормозного расстояния автомобиля, который движется со скоростью 50 км/ч, будет выглядеть следующим образом: (50/10) x (50/10) / 2 = 12,5 метров.
  • В результате теперь мы можем вычислить реальный итоговый тормозной путь автомобиля. Так, при нормальном (не резком, а обычном) торможении итоговый тормозной путь составит около 40 метров. При экстренном торможении – не менее 28 метров. 

 

Примечание: Обратите внимание, что если скорость автомобиля будет выше всего в два раза, его итоговый тормозной путь увеличится в четыре раза!!!

 

Смотрите также: Основные принципы работы тормозного механизма автомобиля [Принцип работы и элементы тормозной системы]

 

То есть мнение о том, что при увеличении скорости автомобиля в два раза тормозной путь увеличивается только в два раза, – это чистый воды миф среди многих автолюбителей. Так что имейте это в виду, когда садитесь за руль. Самое удивительное, что об этом не знают даже многие опытные водители. 

 

Пример расчета тормозных и остановочных расстояний

Скорость, в км / ч

Путь, пройденный автомобилем

во время реакции водителя, в метрах

Тормозное расстояние, в метрах

(с момента нажатия педали тормоза

до полной остановки машины)

Итоговый тормозной путь, в метрах

25

7,5

6,25

13,75

50

15

25

40

100

30

100

130

150

45

225

265

200

60

400

460

 

Какие факторы влияют на торможение и тормозной путь?

 

Решающим значением для длины тормозного пути, конечно же, является скорость автомобиля, с которой он движется по дороге. Также на тормозной путь влияет качество установленной на машину тормозной системы. В том числе важную роль, несомненно, играет и состояние дороги (снег, лед, качество асфальта/бетона, трещины в дорожном покрытии, листья, лужи и т. п.). И само собой, не стоит забывать о состоянии шин автомобиля. Ведь в определенных случаях изношенная резина сильно увеличит тормозной путь автомобиля, так как не сможет передавать нормальную тормозную способность дорожному покрытию в отличие от новых шин, имеющих нормальное сцепление с дорогой. 

Также ясно, что на мокрой поверхности тормозное расстояние машины больше, чем на сухом асфальте. 

 

Не стоит забывать и об уровне подготовки водителя. Особенно важна, как мы узнали, для итогового тормозного пути скорость реакции водителя на дорожную ситуацию, требующую остановки автомобиля. Но скорость реакции за рулем зависит не только от опыта вождения. Например, знаете ли вы, что когда вы садитесь за руль в сонном состоянии (не выспались, устали или долго находились за рулем), то скорость реакции может замедлиться почти в два раза по сравнению со скоростью реакции хорошо отдохнувшего водителя. 

В целом же на скорость принятия решения за рулем (скорость реакции) влияет много факторов: возраст водителя, алкогольное или похмельное состояние, употребление определенных медикаментов и в целом состояние здоровья. Так, при многих хронических заболеваниях скорость реакции многих водителей существенно снижается. Следовательно, все эти факторы серьезно влияют на тормозной путь автомобиля. 

 

Смотрите также: Тормозной путь автомобиля: Все что нужно знать

 

То же самое касается и отвлечения внимания из-за смартфонов, которыми так любят пользоваться за рулем многие водители, несмотря на строгий запрет согласно нашему действующему законодательству.

 

Как мы уже сказали, на тормозной путь также влияет время отклика тормозной системы автомобиля на нажатую педаль тормоза. Особенно это касается старых автомобилей. Современные же, как правило, оснащены уже новым поколением тормозов, которые мгновенно активируются за счет максимального тормозного давления, как только вы резко ударите ногой по педали тормоза (например, при экстренном торможении). Эта технология позволила существенно сократить итоговый тормозной путь современных машин. 

 

Как повысить безопасность при управлении автомобилем?

 

Не зря основное правило вождения гласит о том, что водитель должен держать на дороге достаточную дистанцию до других автомобилей, чтобы оставалось пространство для экстренного торможения и для того, чтобы не спровоцировать ДТП. Но, с другой стороны, вы не должны держать дистанцию между автомобилями слишком большой. Помните, что все должно быть в меру. Вот некоторые правила вождения от экспертов:

 

  • В городском движении: Держите расстояние до других автомобилей около 15 метров. 
  • На автомагистралях, шоссе и проселочных дорогах: При скорости движения около 100 км/ч держите дистанцию примерно 50 метров. При плохой видимости или на скользкой дороге дистанция до других машин должна быть увеличена в два раза. Например, при скорости в 100 км/ч на скользкой дороге держите расстояние до впереди идущей машины минимум в 100 метров. 

неужели тормозной путь не зависит от массы авто?

Друзья, в прошлом выпуске я утверждал, что тормозной путь автомобиля не зависит от его массы. Большинство водителей считают, что зависит, и я объяснил, откуда берется это представление. В этой статья я докажу справедливость своего утверждения, прибегнув к физическим понятиям.

Подчеркну, что речь идет о кратчайшем, экстренном, то есть минимально возможном тормозном пути. То есть о тормозном пути при торможении на грани блокировки колес. В современных машинах при таком торможении срабатывает АБС (антиблокировочная система тормозов), а классические машины либо срываются в «юз», либо остаются на грани «юза», в зависимости от действий водителя.

Сначала докажу это «на пальцах». Утяжеляя машину, мы, с одной стороны, увеличиваем ее инертность и осложняем торможение. С другой стороны, мы сильнее прижимаем шины к дороге, увеличиваем сцепление шин с дорогой и повышаем тормозные возможности машины. Эти два эффекта компенсируют друг друга в равной степени, и, в конечном итоге, масса не влияет на длину тормозного пути.

Что такое «масса»?

Для интерсующихся приведу физико-математическое доказательство и вначале кратко расскажу о понятии «масса». Массы в природе две: инертная и гравитационная. Есть, правда, еще и третий вариант – Фелипе Масса, пилот Формулы 1, уже который год выступающий за Ferrari, но сейчас не об этом 🙂

Инертная масса

Инертная масса mи – масса, которая «отвечает» за сопротивление движению тела. Чем тяжелее тело, тем сложнее привести в его движение или остановить, если оно движется.

В механике об этом говорит 2-й закон Ньютона:

a = F/mи

то есть ускорение (замедление) тела пропорционально действующей на него силе и обратно пропорционально инертной массе тела. Или в более привычной формулировке этот закон выглядит как

F = mи a

Инертная масса осложняет торможение

Это как раз то, о чем думает большинство водителей: чем тяжелее машина, тем сложнее ее остановить (а также и разогнать) и, якобы, тем длиннее тормозной путь. Остановить машину действительно сложнее, не спорю, но тормозной путь есть возможность сохранить — для этого нужно лишь затратить больше энергии. В этом нам поможет второе понятие массы.

Гравитационная масса

Гравитационная масса mг – масса, которая «отвечает» за взаимное притяжение тел, в частности, за притяжение тел к Земле. Чем тяжелее тело, тем больше сила тяготения и тем сильнее тело давит на опору (пол, дорогу и т.д.).

А об этом в механике говорит закон всемирного тяготения Ньютона:

F = G mг1 mг2/r2

Или, по-русски, сила притяжения двух тел пропорциональна массам (гравитационным) этих тел и обратно пропорциональна квадрату расстояния между ними.

Эта формула упрощается для тела в поле тяготения Земли:

F = mг g

где mг – гравитационная масса тела, а g – ускорение свободного падения, равное 9,81 м/с2

Гравитационная масса помогает торможению

Применительно к разговору о тормозном пути это означает, что чем тяжелее машина, тем сильнее она давит на колеса, тем лучше прижимает их к дороге и тем лучше сцепление шин с дорогой. Ведь, согласно закону Кулона, сила сила трения покоя (в нашем случае — сила сцепления шин с дорогой, она же – «держак» на гоночном жаргоне) пропорциональна весу тела N:

Fтр = k N = k mг g

где mг – гравитационная масса машины, k – коэффициент сцепления шин с дорогой, g – ускорение свободного падения.

Тогда, чем больше масса автомобиля, тем выше сила сцепления шин с дорогой и тем сложнее тормозам заблокировать колеса и пустить машину в «юз» (ну или включить АБС, если она есть).

Одна масса мешает, другая — помогает. Что победит?

В итоге, инертная масса увеличивает инерцию машины, а гравитационная масса улучшает сцепление шин с дорогой и тормозной потенциал машины. Одно удлиняет тормозной путь, а другое пытается укоротить его. Что же победит?

Нам поможет Закон сохранения энергии

На языке физики процесс торможения выглядит как закон сохранения энергии:

mи v2/2 = Fтр s

т.е. кинетическая энергия машины с инертной массой mи и скоростью v при торможении переходит в тепло за счет работы силы трения Fтр, которая затрачивается на замедление машины на участке пути длиной s (собственно, тормозной путь).

Машина тормозит не тормозами, а шинами

Как я уже писал выше, сила трения Fтр равна kmг g – произведение коэффициента трения k, гравитационной массы mг и ускорения свободного падения g. И сразу вопрос: о какой силе трения идет речь? О силе трения колодок о тормозной диск? Или о силе трения шины о дорогу, о «держаке»? Вообще, первопричина торможения – сила трения колодок о диски. Но она не может превышать силу трения между шиной и дорогой: в этом случае шины начинают скользить, и, либо включается АБС, либо машина идет в «юз». После чего любое усиление нажатия на тормоз не дает выигрыша в торможении, и машина продолжает тормозить за счет трения шин о дорогу. Поэтому для случая экстренного торможения нужно считать, что сила трения колодок о диски равна силе сцепления шин с дорогой. И тогда k — коэффициент сцепления шин с дорогой, если шины на грани скольжения, или это коэффициент скольжения шин о дорогу, если колеса заблокированы, и машина тормозит юзом.

Тогда подставим значения силы сцепления Fтр = k mг g в закон сохранения энергии:

mи v2/2 = k mг g S

Инертная и гравитационная массы противодействуют друг другу в равной степени

А теперь ключевой момент! Еще Ньютон доказал, а Эйнштейн в свое время постулировал, что инертная и гравитационные массы равны! На сегодняшний день это проверено многократными экспериментами с высокой степенью точности. Эти массы имеют абсолютно разный физический смысл, но в килограммах это всегда одно и то же!

И тогда заменяем инертную и гравитационную массы на «просто массу»:

m v2/2 = k m g S

Теперь массы можно успешно сократить, и останется:

v2/2 = k g S

Отсюда получаем тормозной путь, не зависящий от массы:

S = v2/(2 k g)

где v – скорость движения машины до начала торможения, k – коэффициент сцепления шин с дорогой, g – ускорение свободного падения.

Еще раз смысл: с одной стороны, масса увеличивает инертность машины и создает препятствие тормозам. С другой стороны, масса увеличивает сцепление шин с дорогой и помогает тормозам. Эти два эффекта компенсируют друг друга в равной степени, и, в конечном итоге, масса не влияет на длину тормозного пути.

Скорость зависит только от водителя, g – постоянна, а коэффициент сцепления k зависит от состава резины протектора шины и от качества дорожного покрытия. Выходит, тормозной путь зависит от скорости, качества шины и качества дороги. При этом под качеством шины понимается именно состав резины. А от ширины профиля шины и площади пятна контакта сила сцепления шины с дорогой не зависит, как и не зависит тормозной путь.

Тормоза важны

Поговорим о тормозах. Размеры тормозных дисков, материалы колодок и прочее устройство тормозных механизмов важны для машины, но не могут влиять на тормозной путь напрямую, поскольку он ограничивается сцеплением шин с дорогой. Но хочу отменить следующее. Каждые тормозные механизмы расчитаны на погашение определенной кинетическиой энергии, которая пропорциональна массе и квадрату скорости. Обычно запас тормозов расчитывают так, чтобы даже Форд Фокус остановился с мешком картошки в багажнике со 100 км/ч за те же 40 метров, что и без мешка. Но вот ежели вы в машину загрузите лишних 500 кило, будьте готовы к тому, что ваши тормозные механизмы, рассчитанные под меньшую массу, перегреются и не справятся с задачей, и проедете вы куда больше прежних 40 метров.

Или еще пример. Можно взять Жигули со штатными тормозными дисками и колодками и поставить на нее гоночные слики. А что, на Формулах 1 как раз шины 13-дюймового диаметра, аккурат подойдут 🙂 Конечно, придется серьезно переделать саму машину, но это сейчас не столь важно. Так вот, слики имеют почти вдвое больший коэффициент сцепления с дорогой, а значит для торможения юзом на тормоза Жигулей ляжет нагрузка вдвое больше обычной. И вариантов развития событий тоже два: либо тормоза перегреются с первой же попытки, либо вовсе не смогут довести колеса до грани блокировки… И то, и другое означает для нас увеличение тормозного пути (по сравнению с тормозным путем на этих же сликах и гоночными тормозами) даже для пустой машины. А если ее еще и догрузить как следует, то ситуация еще более усугубится, и тормозной путь таких Жигулей еще как будет зависеть от массы авто.

Таким образом, мы можем говорить о независимости тормозного пути от массы машины, если она соответствует общепринятым нормам безопасности: на машине с загрузкой, не превышающей допустимую производителем, штатные тормоза должны быть способны заблокировать колеса (или включить АБС) на штатных шинах.

Однако главное при торможении — шины

Выходит, и Жигули, и Ferrari затормозят с примерно одинаковым тормозным путем, если тормоза у всех исправны, а на колеса установлены одни и те же шины. Возможна разница за счет разного времени срабатывания тормозной системы, а также за счет разных алгоритмов торможения водителя и АБС. Но эта разница будет куда меньше по сравнению с тем, когда одни и те же Жигули (или Ferrari) будут тормозить сначала на Michelin, а потом на отечественной Каме. Так что главное при торможении — шины!

Выше я уже написал, что в случае торможения на грани скольжения шин под k понимается коэффициент сцепления, а в случае торможения юзом при заблокированных колесах k — коэффициент скольжения шин по дороге. Известно, что трение скольжения всегда меньше трения покоя (сцепления), примерно на 10-15%. Соответственно, машина, тормозящая юзом, как правило, проходит на 10-15% больший путь до полной остановки по сравнению с машиной, тормозящей на грани скольжения. АБС не допускает блокировки колес, поэтому машины с АБС при нажатии тормоза «в пол» тормозят всегда на грани скольжения. А машины без АБС при торможении «в пол» сразу же уходят в юз. Хотя, при должном навыке водитель и без АБС может правильно дозировать усилие на педали и тормозить на грани скольжения. Например, машины в Формуле 1 не оснащены АБС, и пилоты тормозят на грани скольжения, а уход в юз считается ошибкой. Из написанного следует, что при одних и тех же шинах машина с АБС будет тормозить короче, чем машина без АБС юзом, но это справедливо только для гладких и твердых дорог. На рыхлых и неровных покрытиях машины с АБС проигрывают в тормозном пути машинам без АБС.

Кстати, не стоит сравнивать тормозные пути седана и фуры. Это не всегда корректно, поскольку там могут быть конструктивно разные тормоза (у грузовиков даже бывает не гидравлическая, а пневматическая тормозная система с огромной задержкой в срабатывании) и разного качества шины. Лучше всего сравнивать «яблоки с яблоками», то есть одну и ту же машину с разной степенью загрузки. Подробнее об этом читайте в ответе на вопрос гостя нашего сайта о влиянии тормозов.

Легковушка и фура тормозят одинаково

Однако, если время срабатывания тормозов у легковушки и фуры одинаково, и стоят схожие по составу шины, то тормозной путь отличаться не должен. Вот видео, которое подтверждает это (правда, я не понимаю по-немецки, но по смыслу именно то :)):

http://www.myvideo.de/watch/7778214/Bremstest_PKW_LKW_VW_T4_gg_Mercedes_Actros

В заключение скажу, что тормозной путь зависит от веса машины (не будем путать вес и массу), а также от массы прицепа без тормозов, от положения руля. Обо всем этом я расскажу в будущих выпусках.

Как это поможет на практике?

А пока — практический смысл этой статьи.

Используйте качественные шины

Помните, машина тормозит не тормозами, а шинами. Если у вас стоят изношенные или дешевые или просто не соответствующие сезону шины, ваш автомобиль тормозит плохо, и хорошие тормоза ему не помогут. Если вы хотите повысить безопасность и улучшить тормозную динамику машины, не нужно делать тюнинг тормозов и ставить дорогущие тормозные диски, колодки и т.п. Поставьте дорогие качественные шины, и тогда ваша жизнь за рулем будет в большей безопасности.

Тюнинг машины требует профессионального подхода

Если же вы решите «обуть» машину в суперцепкие шины — для гонок ли, или для собственной безопасности, имейте в виду, что это уже вмешательство в конструкцию автомобиля, тюнинг. Одними шинами не обойтись — они потребуют для себя мощных тормозов, а подобрать их и грамотно установить — дело крайне важное и непростое. Так что подходите к тюнингу машины серьезно и пользуйтесь услугами профессионалов, ведь такие вещи не терпят самодеятельности.

Маленькая легкая машина не дает преимуществ при торможении

Выбирая машину при покупке не думайте, что маленький городской автомобильчик будет более безопасный по сравнению с минивэном и тем более фурой лишь потому, что легче и, якобы, лучше тормозит. Не лучше он тормозит, а если и лучше, то масса тут ни при чем. Будьте бдительны, если управляете маленьким авто. Особенно, когда едете сзади фуры: не приближайтесь к ней и не думайте, что в случае чего она будет останавливаться долго, а вы то уж точно успеете остановиться… Сохраняйте безопасную дистанцию, независимо от разницы в массах машин.

Сохраняйте самообладание, управляя загруженной машиной

Если вам предстоит путь на машине с пассажирами и полным багажником, будьте бдительны, но не теряйте самообладание при торможении. Да, вам покажется, что торможение стало хуже. Но это лишь потому, что вы привыкли к другому усилию на педали тормоза.Нажимайте на тормоз сильнее обычного, и машина затормозит так, как вам нужно. Но и после разгрузки автомобиля не теряйте голову 🙂 — ведь машина станет более чутко отзываться на нажатие педали тормоза, но это иллюзия: тормозной путь не станет короче!

Не перегружайте машину

У каждой машины есть свое предназначение для использования и своя допустимая нагрузка. Если ее превысить, то шины и тормоза могут перегреться, а то и вовсе испортиться. В любом случае, они не справятся с задачей торможения. Тормозной путь заметно увеличится, и это, как вы понимаете, может привести к ДТП.

Учитесь правильно тормозить

Казалось бы, что тут сложного? Но наш тренерский опыт говорит, что многим водителям не хватает плавности и знаний многих тонкостей в повседневном торможении и, наоборот, маловато резкости в экстренном торможении. В общих чертах я написал об этом в статье «Как правильно тормозить?», а если вас интересует практика, то экстренное торможение вы можете отработать на курсе «Зимняя контраварийная подготовка», а постичь все премудрости грамотного торможения на каждый день — на «курсе МВА для водителя: Мастерство Вождения Автомобиля».

Калькулятор остановочного пути автомобиля • Механика • Онлайн-конвертеры единиц измерения

Калькулятор определяет остановочный путь автомобиля с момента обнаружения водителем опасности до момента полной остановки автомобиля, а также другие параметры, связанные с этим событием, в частности, время восприятия водителем сигнала о необходимости торможения, время реакции водителя, а также расстояние, которое прошел автомобиль во время этих событий. Калькулятор также определяет начальную скорость (скорость до начала торможения) по известной длине торможения (длины тормозного пути) с учетом дорожных условий. Как и все остальные калькуляторы, этот калькулятор не следует использовать в судебных процессах и при необходимости получения высокой точности.

Пример 1: Рассчитать расстояние, необходимое для остановки автомобиля, движущегося со скоростью 90 км/ч по мокрой горизонтальной дороге с асфальтобетонным покрытием (коэффициент трения μ = 0,4) если время восприятия водителя 0,5 с и время реакции водителя 0,7 с.

Пример 2: Рассчитать начальную скорость автомобиля, движущегося по дороге с мокрым асфальтобетонным покрытием (μ = 0.4), если длина тормозного пути равна 100 м. Автомобиль движется на спуске с уклоном 10%.

Калькулятор остановочного пути

Входные данные

Начальная скорость

v0м/скм/чфут/смиля/ч

Время восприятия опасности водителем

thpс

Время реакции водителя

thrс

Уклон

σградус%

Движение вверх Движение вниз

Состояние дороги

—Сухой асфальтМокрый асфальтПокрытый снегом асфальтПокрытый льдом асфальт

или Коэффициент трения

μ

Тип привода тормозов

—ПневматическийГидравлический

или Время срабатывания тормозной системы

tbrlс

Выходные данные

Угол крутизны уклона θ= °

Замедление a= м/с²

Время торможения tbr= с

Расстояние, которое проедет автомобиль во время восприятия водителем опасности Shp= м

Расстояние, которое проедет автомобиль во время реакции водителя на опасность Shr= м

Расстояние, которое проедет автомобиль за время задержки срабатывания тормоза Sbrl= м

Тормозной путь Sbr= м

Остановочный путь Sstop= м

Критический угол наклона для заданного коэффициента трения θcrit= °

Критический уклон для заданного коэффициента трения σcrit= %

Определения и формулы

Остановочный путь

Остановочный путь — это расстояние, которое проходит автомобиль с момента, когда водитель видит опасность, оценивает ее, принимает решение остановиться и нажимает на педаль тормоза и до момента полной остановки автомобиля. Это расстояние является суммой нескольких расстояний, которые проходит автомобиль в то время, как водитель принимает решение, срабатывают механизмы тормозной системы и происходит замедление движения до полной остановки.

где shr — расстояние, которое проедет автомобиль во время восприятия и оценки водителем ситуации, shr — расстояние, которое проедет автомобиль во время во время реакции водителя на ситуацию, sbrl — расстояние, которое проедет автомобиль во время задержки срабатывания тормозов, и sbr — тормозной путь.

Расстояние, которое пройдет автомобиль во время восприятия и оценки водителем ситуации

Расстояние человеческого восприятия ситуации — это расстояние, которое пройдет автомобиль в то время, пока водитель оценивает опасность и принимает решение уменьшить скорость и остановиться. Оно определяется по формуле

где shp расстояние человеческого восприятия в метрах, v скорость автомобиля в км/ч, thp — время человеческого восприятия в секундах и 1000/3600 — коэффициент преобразования километров в час в метры в секунду (1 километр равен 1000 метров и 1 час равен 3600 секундам).

Расстояние, которое пройдет автомобиль во время реакции водителя

Расстояние реакции водителя — это расстояние, которое пройдет автомобиль пока водитель выполняет решение остановить автомобиль после оценки опасности и принятия решения об остановке. Оно определяется по формуле

где shp — расстояние реакции водителя с метрах, v — скорость автомобиля в км/ч и thr — время реакции водителя в секундах.

Расстояние, которое пройдет автомобиль во время срабатывания тормозной системы

Расстояние, которое пройдет автомобиль во время срабатывания тормозной системы, зависит от типа тормозной системы, установленной на автомобиле. Почти на всех легковых автомобилях и малотоннажных грузовых автомобилях используются гидравлическая тормозная система. На большинстве большегрузных автомобилей используются тормоза с пневматическим приводом. Задержка срабатывания пневматических тормозов приблизительно равна 0,4 с, а гидравлических (жидкость несжимаема!) 0,1–0,2 с. Общая задержка срабатывания тормозной системы измеряется как время от момента нажатия на педаль тормоза, в течение которого замедление становится устойчивым. Оно состоит из задержки срабатывания тормозной системы и времени установления постоянной величины замедления движения. В тормозной системе с пневматическим приводом воздуху необходимо время, чтобы пройти по тормозным магистралям. С другой стороны, в гидравлическом приводе задержек практически не наблюдается, и он работает в два—пять раз быстрее, чем пневматический.

Расстояние, которое пройдет автомобиль во время срабатывания тормозной системы, определяется по формуле

где sbrl — расстояние в метрах, которое пройдет автомобиль во время срабатывания тормозной системы, v — скорость движения автомобиля в км/ч, tbrl — время срабатывания тормозной системы в секундах.

Замедление

Для упрощения расчетов предположим, что автомобиль движется с постоянным ускорением или замедлением, которое определяется по известной из курса элементарной физики формуле равноускоренного или равнозамедленного движения

где a — ускорение, v — начальная скорость, v0 — конечная скорость и t — время.

Тормозной путь автомобиля

Тормозной путь автомобиля — это расстояние, которое проходит автомобиль с момента полного нажатия на педаль тормоза до момента полной остановки. Это расстояние зависит от скорости автомобиля перед началом торможения и от коэффициента трения между шинами и дорожным покрытием. В этом калькуляторе мы не учитываем другие факторы, влияющие на тормозной путь, например, сопротивление качению шин или лобовое сопротивление воздуха

В результатах исследования1, в котором коэффициент трения определялся путем измерения замедления, определено, что антиблокировочная тормозная система (АБС) влияла на коэффициент трения таким образом: он увеличивается с увеличением скорости при использовании АБС и уменьшается, если АБС не используется. В этом исследовании также подтверждается, что на коэффициент трения между шинами и дорожным покрытием влияет температура и интенсивность дождя.

Вывод зависимости тормозного пути от скорости и трения с использованием второго закона Ньютона

Коэффициент трения определяется как отношения силы трения к силе нормального давления, прижимающей тело к опоре:

или

где Ffr — сила трения, μ коэффициент трения и Fnorm — сила реакции опоры.

Действующая на тело нормальная сила реакции опоры определяется как составляющая силы реакции, перпендикулярная к поверхности опоры тела. В простейшем случае, когда тело находится на плоской горизонтальной поверхности, нормальная сила равна весу этого тела:

где m — масса тела и g — ускорение свободного падения. Эта формула выведена из второго закона Ньютона:

В более сложном случае, если тело расположено на наклонной плоскости, нормальная сила рассчитывается как

где θ — угол наклона между плоскостью поверхности и горизонтальной плоскостью. В этом случае нормальная сила меньше веса тела. Случай наклонной поверхности мы рассмотрим чуть позже.

В случае же горизонтальной поверхности, если коэффициент трения между телом и поверхностью равен μ, то сила трения равна

В соответствии со вторым законом Ньютона, эта сила трения, приложенная к движущемуся телу (автомобилю) приводит к возникновению пропорционального ей замедления:

или

Теперь, в соответствии с уравнением ускоренного (замедленного) движения имеем

Из курса элементарной физики известно, что при равнозамедленном движении с постоянным замедлением, если конечная скорость равна нулю, то тормозной путь определяется уравнением

Это уравнение можно переписать в более удобной форме с использованием преобразования скорости в км/час в м/с:

Подставляя в это уравнение a = μg, получаем формулу тормозного пути:

где скорость v задается в км/час, а ускорение силы тяжести g в м/с².

Решая это уравнение относительно v, получаем:

Аналогичную формулу для определения тормозного пути можно получить с помощью энергетического метода.

Вывод зависимости тормозного пути от скорости и трения с помощью энергетического метода

Теоретическое значение тормозного пути можно найти, если определить работу по рассеиванию кинетической энергии автомобиля. Если автомобиль, движущийся со скоростью v, замедляет движение до полной остановки, работа тормозной системы Wb, требуемая для полного рассеяния кинетической энергии автомобиля Ek, равна этой энергии:

Кинетическая энергия движущегося автомобиля Ek определяется формулой

где m — масса автомобиля и v — скорость движения автомобиля перед началом торможения.

Работа Wb, выполненная тормозной системой, определяется как

где m — масса автомобиля, μ — коэффициент трения между шинами и дорожным покрытием, g — ускорение силы тяжести и sbr — тормозной путь, то есть расстояние, которое прошел автомобиль от начала торможения до полной остановки.

Теперь, с учетом того, что Ek = Wb, имеем:

или

Скорость автомобиля до начала торможения является наиболее важным фактором, влияющим на величину остановочного пути. Другими, менее важными, факторами, влияющими на остановочный путь, являются время оценки водителем ситуации, время реакции водителя, скорость работы тормозной системы автомобиля и состояние дороги.

Время торможения

Из курса элементарной физики известно, что средняя скорость при равноускоренном движении равна полусумме начальной и конечной скорости:

С учетом, что конечная скорость равна нулю, время торможения определяется в калькуляторе как

Движение вверх и вниз по уклону

Силы, действующие на автомобиль на уклоне: Fg — сила тяжести (вес автомобиля), Fgd — скатывающая вниз составляющая веса автомобиля, Ffr — сила трения, действующая параллельно поверхности дорожного полотна с уклоном, Fgn — нормальная составляющая веса автомобиля, направленная перпендикулярно поверхности дороги, и Fnr — сила реакции опоры, равная нормальной составляющей веса автомобиля.

Когда водитель нажимает на педаль тормоза, замедляющий движение автомобиль может быть представлен в виде тела на поверхности с углом наклона θ (см. рисунок выше). Для простоты мы будем рассматривать только две силы, действующие на автомобиль, находящийся на уклоне. Это вес автомобиля и сила трения. Автомобиль, движущийся с начальной скоростью, замедляет движение, если сила трения, действующая параллельно дорожному полотну, больше, чем скатывающая сила, являющаяся составляющей силы тяжести, которая также параллельна дорожному полотну. Если начальная скорость автомобиля равна нулю, он в этой ситуации остается на месте при условии, что угол уклона меньше критического (об этом — ниже).

В то время, как сила тяжести Fg стремится скатывать автомобиль вниз, сила трения Ffr сопротивляется этому движению. Чтобы автомобиль мог в этой ситуации остановиться, сила трения должна превышать скатывающую составляющую силы тяжести Fgd.

В то же время, если сила трения превышает скатывающую составляющую силы тяжести, автомобиль будет двигаться вниз с постоянным ускорением и его тормозная система будет неспособна его остановить. Это может произойти, если угол наклона (уклон) дорожного полотна слишком велик или коэффициент трения слишком мал (вспомним как ведет себя автомобиль с обычными шинами на уклоне, если он покрыт коркой льда!).

По определению коэффициента трения, можно записать уравнение для силы трения:

или

Скатывающая составляющая силы тяжести:

Результирующая сила Ftotal, действующая на автомобиль на уклоне:

или

Как мы уже отмечали, сила Ftotal должна быть направлена вверх, иначе автомобиль при движении вниз остановить невозможно. В соответствии со вторым законом Ньютона, ускорение (точнее, замедление) автомобиля, движущегося под действием силы Ftotal, определяется как

Подставляя ускорение в выведенную выше формулу тормозного пути, получаем:

Решая это уравнение для vpre-braking, получим:

Отметим еще раз, что в этих формулах g задается в м/с, v в км/ч и s в метрах. В нашем калькуляторе используются две последние формулы.

Припаркованные и движущиеся по ул. Дивисадеро в Сан-Франциско (Калифорния) автомобили. Уклон дорожного полотна в этом месте равен 31% или 17°.

Уклон

Величина уклона дороги (показателя крутизны склона) равна тангенсу угла плоскости дорожного покрытия к горизонтали. Он рассчитывается как отношение перпендикуляра, опущенного из точки на поверхность (превышения местности) к длине горизонтальной поверхности от начала склона до перпендикуляра (горизонтальному расстоянию). По определению уклона считается, что при движении вверх уклон является положительным, а при движении вниз уклон является отрицательным, когда превышение в действительности является понижением дороги. Уклон дороги σ выражают как угол наклона к горизонтали в градусах или как отношение в процентах. Например, подъёму 15 метров на 100 метров перемещения по горизонтали соответствует уклон, равный 0,15 или 15%. В этом калькуляторе мы используем уклон в процентах, определяемый по формуле

где Δh — превышение местности и d — проекция уклона на горизонталь (см. рисунок выше). Если известен уклон, то угол наклона можно определить по формуле

Критический угол

При увеличении угла наклона дорожного полотна выше определенного значения, называемого критическим углом, движущийся вниз автомобиль затормозить невозможно, так как действующая на него сила трения становится меньше скатывающей силы. Этот критический угол находится из условия

или

или

Из этой формулы можно найти критический угол для данного коэффициента трения, при котором автомобиль не сможет затормозить:

Уклон, выраженный в процентах, определяется по известному углу наклона таким образом:

Пример

В этом примере мы покажем, как использовать формулу для определения тормозного пути. Пусть автомобиль движется с начальной скоростью vpre-braking = 90 км/ч вниз по уклону σ = 5% по мокрому асфальту (коэффициент трения μ = 0,4). Нужно определить тормозной путь. Для расчетов используем выведенные выше формулы.

Особые случаи

Нажмите на соответствующую ссылку, чтобы посмотреть как работает калькулятор в особых режимах:

Литература

  1. Hartman, J 2014, Effects of velocity, temperature And rainfall on the friction coefficient of pneumatic tyres And bitumen roads, Doctor of Philosophy (PhD), Aerospace, Mechanical And Manufacturing Engineering, RMIT University PDF 48 MB
  2. Wikibooks. Fundamentals of Transportation

Формула нахождения тормозного пути


Тормозной путь — калькулятор, формула и расчет онлайн

Калькулятор тормозного пути позволит оценить тормозной путь автомобиля, движущегося с заданной скоростью. Для использования укажите тип дорожного покрытия, на котором тормозит автомобиль и скорость, при которой начинается торможение. Калькулятор рассчитает сколько метров пройдет автомобиль при торможении.

Калькулятор тормозного пути

Формула тормозного пути

Формула для нахождения тормозного пути

Формула для нахождения тормозного пути применяется в подразделениях ГИБДД. Именно она используется в нашем калькуляторе. В этой формуле:

S — тормозной путь,

Кт — тормозной коэффициент (для легкового автомобиля равен 1),

V — скорость автомобиля,

Kсц — коэффициент сцепления.

Понятия и пояснения

Тормозной путь — это путь, который проходит автомобиль с момента, когда сработал тормозной механизм до полной остановки автомобиля. На него влияют:

  • состояние и тип дорожного покрытия,
  • состояние шин автомобиля,
  • начальная скорость автомобиля,
  • масса автомобиля,
  • исправность тормозной системы.

Остановочный путь — путь с момента обнаружения опасности до полной остановки автомобиля. Понятно, что тормозной путь входит в остановочный. Кроме того в остановочный путь входят:

  1. путь, который проехал автомобиль с момента обнаружения опасности до нажатия на педаль тормоза;
  2. путь, пройденный автомобилем за время срабатывания тормозной системы.

Первый параметр зависит от множества факторов, определяющим из которых является времени реакции водителя. По результатам многочисленных экспериментов, оно может меняться от 0,3 до 1,5 секунды. В среднем можно считать время реакции водителя равное 1 секунде. Кроме этого существует понятие «нормативное время восприятия сложной ситуации» равное 0,8 секунды. Также установлено, что время реакции у женщин, при возникновении сложной дорожной ситуации может достигать 2,5-3 секунд, тогда как у мужчин 1,5-2 секунды. Кроме этого на время реакции влияет:

  • опыт водителя,
  • его эмоциональное состояние,
  • возраст,
  • время суток и погодные условия,
  • прием медикаментов,
  • состояние алкогольного или иного опьянения,
  • место возникновения опасной ситуации.

Время срабатывания тормозной системы зависит от ее типа и технического состояния. Тормозная система с гидравлическим приводом срабатывает за 0,2 – 0,3 секунды, с пневматическим за 0,5 –0,6 секунд.

Ваша оценка

[Оценок: 94 Средняя: 4.6]

Расчет тормозного пути Автор admin средний рейтинг 4.6/5 — 94 рейтинги пользователей

calculat.ru

что это и как рассчитать

Подробности
Категория: Блог о продаже автомобиля
Создано: 18 февраля 2020
Просмотров: 170

Всем водителям не следует забывать о том, что при нажатии на тормоз авто мгновенно не останавливается. Для этого нужно некоторое время, зависимое от многих факторов. Согласно ПДД, необходимо соблюдать безопасную дистанцию между своим и идущим впереди авто, чтобы при надобности успеть затормозить. Для расчета данной дистанции нужно знать, что такое тормозной путь. Кроме того, большинство людей путают остановочный и тормозной путь.

Тормозной путь: что это

Независимо от опытности и профессионализма водителя, за рулем случаются ситуации, когда нужно оперативно остановить авто:

  • На дороге неожиданно возникает животное или люди;
  • Возникает поломка авто;
  • Другие водители нарушают ПДД, создавая аварийную обстановку;
  • Происходят непредвиденные ситуации: препятствия, неровная дорога и т.д.

Чтобы остановить авто, водители пользуются тормозом. Тормозной путь транспорта – это отрезок, преодолеваемый машиной за время при срабатывании тормозной системы до достижения нулевой скорости.

Читайте также: Срочный выкуп аварийных авто

Факторы, влияющие на тормозной путь

Отрезок торможения разный, в зависимости от ситуации и условий. Факторы, воздействующие на данное расстояние, разделяются на 2 категории:

  1. Факторы, зависящие от автомобилиста;
  2. Факторы, не зависящие от водителя.

Состояние дорог и погода – это условия, на которые не влияет тот, кто сидит за рулем авто. Логично, что в снежную погоду необходимо больше времени, чтобы затормозить машину, чем в сухую. На отрезок торможения также влияет и покрытие трассы. На гладкой дороге, сделанной без включения камня, при торможении отрезок, который проедет транспорт, увеличится.

На заметку! На трассе, покрытой ямами, тормозной отрезок будет маленьким.

Это объясняется тем, что автомобилисты на плохих дорогах развивают небольшую скорость. Больше факторов, зависящих от водителя:

  • Устройство и работоспособность тормозной системы. Важно исправное состояние машины, включая и тормоза, не изношенность колодок, а также оптимальное давление в шинах;
  • Скорость. При малой скорости отрезок торможения уменьшается, и наоборот большая скорость приведет к увеличению расстояние, которое потребуется транспортному средству для остановки;
  • Используемые шины. Износ протектора должен быть оптимальным, а используемая резина должна быть по погоде. Довольно часто многие аварийные ситуации происходят из-за несвоевременной замены шин и из-за езды на некачественной, изношенной резине, которая не способна обеспечить необходимые характеристики и безопасность при езде;
  • Наличие ABS-системы. Такая система на сухом асфальте помогает быстро останавливать авто, в гололед она позволяет сохранять управление, но при этом увеличивается дистанция торможения;
  • Загрузка машины. Легче остановить легкий автомобиль, чем транспорт, который хорошо загружен. Нагруженная машина тормозит на более длинном отрезке;
  • Трезвость водителя. У трезвого человека более быстрая реакция на изменение ситуации на трассе, поэтому при необходимости он быстрее затормозит машину. Поэтому стоит отказываться от алкоголя, планируя сесть за руль. Это позволит избежать аварий и обезопасит людей;
  • Отсутствие отвлекающих моментов при езде. Часто медленная реакция водителя связывается с тем, что он не смотрит за дорогой и отвлекается. В настоящее время зачастую внимание отвлекает сотовый телефон и другие гаджеты, которые сводят людей с ума, не позволяя отказываться от них даже за рулем. Из-за замедленной водительской реакции увеличивается тормозной отрезок.
Формула для вычисления тормозного пути

Порой нужно рассчитывать тормозной отрезок, к примеру, в следующих условиях:

  • криминалистическая экспертиза;
  • испытания транспорта;
  • проверка работоспособности тормозной системы после ее доработки.

Чтобы произвести подобное вычисление, применяется формула:

Sторм = Кэ * V * V/(254 * Фс), где: S торм. – тормозной отрезок; V – скорость транспорта; Кэ – коэффициент торможения; Фс – коэффициент сцепления, который бывает различным. Так:

  • При мокрой дороге – 0,4;
  • При сухой – 0,7;
  • При гололеде – 0,1;
  • При снеге – 0,2.
Как тормозной путь зависит от дороги?

Коэффициент торможения – это постоянная величина, равняющаяся в основном единице. Рассмотрим пример. Автомобиль едет в летний сезон со скоростью 70 км/ч по сухому асфальту. Нужно вычислить тормозной путь.

S = 1*70*70/(254*0,7) = 28 метров – это и будет отрезок торможения.

Необходимо знать, что тормозной отрезок авто прямо пропорционален квадрату ее скорости. Так, при двойном увеличении скорости, к примеру, с 30 км/ч до 60 км/ч, отрезок торможения увеличился в 4 раза.

Отличие тормозного и остановочного пути

Остановочный и тормозной пути – это различные понятия, но их часто приравнивают и путают. Под остановочным путем понимается расстояние, прошедшее автомобилем после осознания водителем необходимости остановиться до достижения транспортом нулевой скорости. Тормозным путем называется отрезок, который проходит авто от срабатывания тормозов до остановки. Так, остановочный отрезок содержит не только отрезок торможения, но и отрезок, прошедший машиной, пока водитель реагировал на сложившуюся ситуацию.

Вычисление полного времени остановки и итогового тормозного пути

В итоге, величина данного пути содержит не просто дистанцию торможения, но и расстояние водительской реакции. Для вычисления расстояния, пройденного машиной за время водительской реакции, нужно использовать такую формулу:

S реакции = V/ 10*3, где V – скорость машины.

Так, общий тормозной путь станет равным сумме 2-х величин: тормозного пути и отрезок водительской реакции. S итог = Sреакции + Sторм.

Вспоминаем об авто, двигающемся в летнее время со скоростью 70 км/ч по сухой дороге, вычисляем расстояние реакции.

S реакции = 70/10 * 3 = 21 метр

Теперь, когда известно, что тормозной путь — 28 метров, а дистанция реакции – 21 метр, можно вычислить его общее значение: S итог = 28+ 21 = 49 метров

Под полным временем остановки понимается временный промежуток, за который автомобиль проходит общий путь торможения. Это время объединяет время, которое тратится на тормозной отрезок, и время водительской реакции.

Он вычисляется по формуле , где:

  • tp — время водительской реакции;
  • tc — время, когда срабатывает тормозной привод;
  • vo — начальная скорость торможения;
  • tH — время увеличения тормозных сил;
  • g — ускорение свободного падения;
  • Kэ — коэффициент эффективности торможения;
  • φ — коэффициент продольного сцепления колес авто с дорогой.

 

Важно! Общепринятой нормой времени водительской реакции считается 1 секунда. Общий остановочный отрезок включает расстояние водительской реакции и отрезок торможения. На данные величины воздействуют некоторые факторы. Для сокращения величины итогового значения нужно следовать скоростному режиму, проверять исправность авто, быть за рулем в трезвом состоянии и учитывать загрузку машины. Не стоит забывать, что многие ситуации и обстоятельства зависят от самого человека. Поэтому нужно самостоятельно проверять, не нарушать и быть внимательным, чтобы избегать неприятных случаев.

Понравилась статья?

Расскажи друзьям

Читайте также
Порядок и стоимость переоформления автомобиля

Транспортные средства юридических лиц и индивидуальных предпринимателей регистрируются по месту государственной регистрации этих юридических лиц и индивидуальных предпринимателей. Допускается регистрация транспортных средств юридических лиц по месту нахождения их филиалов, представительств и других обособленных подразделений.

Подробнее…
Особенности оформления купли-продажи автомобиля

Переход права собственности на транспортное средство предполагает выполнение некоторых бюрократических процедур и соблюдение ряда формальностей.

Подробнее…
Продал вторую машину-плати налоги

Многие автолюбители даже не подозревают, что, продав два или более авто в течение года, они обязаны подать декларацию в налоговую инспекцию. При этом, если Вы продали второй автомобиль дороже, чем купили, то обязаны заплатить налог с суммы продажи.

Подробнее…
Как продать машину без снятия с учета

Каким образом реализовать дорожное транспортное средство, не снимая с учета? Решение этой проблемы волнует многих автовладельцев.

Подробнее…
Договор купли-продажи автомобиля юридического лица физическому

На данный момент услугами рынка по продаже автомобилей пользуются не только частные лица, но и компании, так как они нуждаются в регулярном обновлении рабочих автомобилей.

Подробнее…
Как грамотно оформить договор купли-продажи автомобиля

При продаже автомобиля очень важно юридически грамотно оформить договор купли-продажи. Действующее законодательство регламентирует определённые правила проведения сделки, учитывая интересы и продавца и покупателя.

Подробнее…
Как избежать штрафа за тонировку авто

Если Вы любите затонировать свой автомобиль вкруговую, то данный текст именно для вас.

Как не получить штрафные санкции за подобное?

Подробнее…
Как продать автомобиль, полученный в наследство

После получения автомобиля по наследству, большинство людей задумываются о его продаже, причин этому достаточно много, возможно автомобиль старый, он вас не устраивает, вы хотите от него избавится или хотите вложить в какие-то внутренние инвестиции.

Подробнее…
Продажа авто при разводе, особенности и «подводные камни»

У нас часто спрашивают, как же продавать машину, если она была приобретена в браке, развод не за горами или уже состоялся.

Подробнее…
Договор купли – продажи и акт приема-передачи автотранспорта

Особо значимым документом при покупке автомобиля является как договор купли-продажи, так и акт приема-передачи автотранспорта. Имея под рукой пример его составления, можно справиться с оформлением акта купли-продажи, не обращаясь за помощью к юристу.

Подробнее…

avtovikyp.by

Как правильно вычислить дистанцию, тормозной и остановочный путь автомобиля: формулы расчета

Формулы расчета остановочного и тормозного пути, а также безопасной дистанции.

В теоретическом экзамене есть вопрос о среднем времени реакции водителя, правильным ответом на который является 1 секунда. Также в билетах ГИБДД имеется вопрос, связанный с безопасной дистанцией. Есть вопросы, касаемые торможения. Но, как говорится, теория – это теория, которая, увы, с практикой, как правило, не имеет ничего общего.

Во-первых, то, что вы учили в билетах, является теорией, основанной на усредненных значениях и различных исследованиях. Фактически же время реакции водителя, остановочный и тормозной путь зависят от многих факторов и не могут быть точно рассчитаны для всех случаев. Тем не менее каждый водитель должен уметь рассчитывать эти параметры хотя бы приблизительно. 

 

Тормозной путь автомобиля

Тормозной путь – это расстояние, которое будет пройдено автомобилем между контактом водителя с педалью тормоза и полной остановкой транспортного средства. Также стоит понимать различия между «нормальным торможением» и «экстренным торможением». В том числе не нужно забывать, что погодные условия влияют на тормозной путь. Если на дороге есть снег, тормозной путь, естественно, увеличивается. 

 

 

Вот формула расчета тормозного пути:

 

(Скорость в км / ч: 10) x (скорость в км / ч: 10) = тормозной путь в метрах

 

Пример расчета: представим, что вы едете со скоростью 50 км/ч по городу и подъезжаете к пешеходному переходу, по которому идут дети. Расчет: (50 км/ч : 10) х (50 км/ч : 10) = 25 (метров). Таким образом, тормозной путь вашей машины составляет 25 метров. Поэтому вы должны учитывать длину тормозного пути, чтобы спокойно своевременно начать тормозить и остановиться перед пешеходным переходом. 

 

Имейте в виду, что при экстренном торможении вы обычно нажимаете педаль тормоза полностью. В этом случае, как правило, тормозной путь сокращается вдвое. Вот формула тормозного пути при экстренном торможении:

 

 

(Скорость в км / ч: 10) x (скорость в км / ч: 10) / 2 = тормозной путь в метрах 

 

Пример расчета: вы едете по городу со скоростью 50 км/ч, и вдруг на дорогу выкатывается мяч, за ним бежит ребенок. Вам нужна экстренная остановка автомобиля. Расчет: (50 км/ч : 10) х (50 км/ч : 10)/2 = 12,5 (метров). Тормозной путь вашей машины при экстренном торможении составит 12,5 метра. 

 

 

Время и путь реакции водителя

Время реакции водителя – это время, которое пройдет с момента обнаружения водителем опасности на дороге до начала принятия мер по ее предотвращению.

 

Путь реакции водителя – это путь, который пройдет автомобиль с момента обнаружения водителем опасности на дороге до нажатия педали тормоза. 

Вот формула расчета пути, который пройдет автомобиль в момент реакции водителя на опасность: 

 

 

(Скорость в км / ч: 10) x 3 = путь реакции в метрах

 

Пример расчета: представим, что вы едете со скоростью 100 км/ч по проселочной дороге и внезапно на дорогу выбегает лось. Расчет: (100 км/ч : 10) х 3 = 30 (метров). То есть, после того как вы среагируете на опасность на дороге, ваша машина проедет примерно 30 метров. Добавьте к этому тормозной путь автомобиля.

 

Внимание: эти правила не являются научно правильными формулами и дают только приблизительное значение!

 

Остановочный путь автомобиля

Остановочный путь – это расстояние, пройденное транспортным средством с момента обнаружения водителем опасности на дороге до полной остановки машины. 

 

 

Если вы хотите рассчитать остановочный путь автомобиля, вы должны добавить к тормозному пути автомобиля путь, пройденный за время реакции водителя. Вот как это можно сделать:

 

(Скорость в км / ч: 10) х 3 + (скорость в км / ч: 10) х (скорость в км / ч: 10)

Первое значение в выражении – это путь реакции водителя, пройденный автомобилем, пока водитель реагирует на опасность на дороге. Второе выражение – это формула расчета тормозного пути. Для того чтобы вычислить остановочный путь транспортного средства, необходимо оба результата сложить вместе. 

 

Пример расчета:  вы едете на своей машине со скоростью  50 км/ч. Расчет: (50 км/ч : 10) х 3 = 15 метров пути проедет машина при реагировании на опасность на дороге (50 км/ч : 10) x (50 км/ч : 10) = 25 метров составит тормозной путь автомобиля. В итоге, сложив оба значения, получаем, что остановочный путь транспортного средства составит 40 метров. 

 

Внимание: эти правила не являются научно правильными формулами и дают только приблизительное значение!

 

Дистанция

  • Три длины автомобиля. Любой, кто путешествует в городских условиях, должен соблюдать дистанцию ​​не менее 15 метров, или три длины автомобиля.
  • Половина спидометра: для безопасной дистанции за пределами населенных пунктов обращайте внимание на скорость автомобиля. Для того чтобы вычислить безопасную дистанцию, разделите на 2 текущую скорость, которую показывает спидометр. В итоге вы получите дистанцию до других автомобилей в метрах. Пример: на скорости 70 км/ч вы должны держаться до впереди идущего автомобиля на расстоянии не менее 35 метров. Причем это касается сухого асфальта в летнее время. 
  • Двойное расстояние: в случае плохой видимости или плохих дорожных условий вы должны удвоить безопасную дистанцию.

1gai.ru

Вот как можно рассчитать тормозной путь: Формула

Как рассчитать расстояние тормозного пути автомобиля.

 

Как быстро автомобиль ускоряется, наверное, знает большинство автовладельцев. Даже если вы не замеряли динамику разгона своей машины, вы наверняка смотрели заводские технические характеристики вашего авто, где обычно автопроизводитель указывает минимально возможное время разгона с 0-100 км/час. Но теперь вопрос: сколько времени нужно, чтобы остановить вашу машину? Вы знаете это? Уверены, что нет. Но, оказывается, рассчитать расстояние тормозного пути можно достаточно легко с помощью простой формулы. Мы расскажем вам, как это делается. 

 

Нет такой вещи во Вселенной или материи, которая может мгновенно остановиться. Также и любой автомобиль, когда вы нажимаете педаль тормоза, не сразу может остановиться. Дело в том, что для того чтобы автомобиль или любой объект в нашем мире остановился, необходимо, чтобы он потерял энергию, которая его движет. В результате у любого автомобиля есть тормозной путь, который он проезжает с момента нажатия педали тормоза до момента полной остановки. Это и есть тормозное расстояние машины.

 

Но на самом деле тормозной путь любого авто зависит не только от его характеристик и тормозной системы, но и от реакции водителя при нажатии педали тормоза. Ведь для того чтобы принять решение о необходимости торможения и нажать педаль тормоза, требуется время, которое хоть и минимально, но достаточно, чтобы машина успела проехать немаленький путь. Особенно это важно при большой скорости движения, где за какие-то доли секунды автомобиль проезжает приличное расстояние. Итак, в итоге, чтобы рассчитать реальную длину тормозного пути, нужно учитывать не только время и расстояние, пройденное автомобилем с момента нажатия водителем педали тормоза до момента остановки машины, но и время, необходимое для принятия решения о торможении. Дело в том, что при принятии решения о торможении мы тратим драгоценные секунды. Вот пример:

 

  • Время отклика: Прежде чем водитель нажмет педаль тормоза, он должен оценить дорожную ситуацию и определить, необходимо ли торможение. Также нужно понять, какое необходимо торможение – полная остановка автомобиля или простое снижение скорости. Обычно, согласно многочисленным исследованиям, большинству водителей для этого требуется около 0,1 секунды. 
  • Время, необходимое для нажатия педали тормоза: После того, как водитель понял, что должен тормозить, необходимо еще примерно 0,8 секунды, для того чтобы переместить ногу с педали газа на педаль тормоза и нажать ее. 

 

Кроме того, даже при нажатии педали тормоза есть еще небольшая потеря времени, связанная с тем, что при нажатии педали тормоза автомобиль, как правило, не начинает резко тормозить. А для того чтобы машина реально начала резко снижать скорость, надо усилить давление на педаль тормоза (пороговое время, необходимое для требуемого тормозного давления в тормозной системе). Также у всех автомобилей разное время отклика на нажатую педаль тормоза. Здесь все, конечно, зависит от конструкции тормозной системы и наличия различной электроники, контролирующей тормоза автомобиля.

 

Смотрите также: Полный привод оказался лучше при торможении, чем привод на два колеса: Видео

 

Вы не поверите, но для того чтобы машина реально начала тормозить после нажатия педали тормоза, необходима еще почти 1 секунда времени. Вы представляете, как это много при движении на большой скорости? За эту лишнюю секунду вы можете проехать очень большой путь. 

 

Что такое формула тормозного пути?

В общем, торможение автомобиля делится на два вида. Например, есть нормальное торможение, а есть экстренное, когда вам нужно резко остановить машину, чтобы избежать аварии.

 

При торможении в повседневной жизни, допустим, если вы хотите остановить автомобиль на светофоре, вы обычно нажимаете педаль тормоза намного плавнее и мягче, чем при необходимости полностью остановить автомобиль на парковке во дворе. В этом случае вы не применяете в машине максимальное тормозное усилие. При таком плавном и мягком торможении, как правило, тормозной путь (тормозное расстояние) увеличивается. Примерное расстояние тормозного пути при нормальном торможении можно рассчитать по следующей простой формуле:

 

(Скорость в км/ч : 10) x (скорость в км/ч : 10) = тормозной путь в метрах

 

При экстренном торможении педаль тормоза, как правило, нажата целиком и с полной силой. Из-за более высокой силы торможения обычно тормозной путь машины сокращается примерно в 2 раза. Поэтому длину тормозного пути можно также вычислить по следующей формуле:

 

(Скорость в км/ч : 10) x (скорость в км/ч : 10) / 2 = тормозной путь в метрах

 

Внимание: Вычисляемый по этим формулам тормозной путь является лишь приблизительным значением и подсказкой для водителей. На самом деле в реальности тормозной путь может быть как меньше, так и больше. Ведь расстояние тормозного пути зависит от навыков и опыта вождения водителя, от технической исправности автомобиля, его конструкции, марки, модели, состояния дорог, состояния протектора резины и многих других факторов, которые напрямую влияют на длину тормозного пути. Но благодаря этим формулам вы примерно сможете высчитать среднюю длину тормозного пути машины при определенной скорости движения. Это позволит вам скорректировать ваш стиль управления автомобилем, а также станет хорошим пособием для водителей-новичков. 

 

Как рассчитать полное время остановки и итоговый тормозной путь?

 

Как мы уже сказали, чтобы рассчитать весь тормозной путь, нужно учитывать потерю времени при принятии водителем решения о торможении (то есть время реакции водителя). Для этого нужно использовать другую формулу, которая обеспечивает более точный приблизительный расчет тормозного расстояния, которое проедет автомобиль в момент принятия решения о необходимости остановки. Вот эта формула:

 

(Скорость в км/ч : 10) x 3 = путь реакции в метрах

 

В итоге, сделав вычисление по вышеуказанным формулам, вы можете вычислить приблизительный итоговый тормозной путь вашего автомобиля при любой скорости движения. Вот пример. Если вы управляете своим автомобилем со скоростью 50 км/ч, то с помощью приведенных формул вычислите следующие значения:

 

  • Тормозной путь при принятии решения о торможении на этой скорости (реакция на дорожную ситуацию + принятие решения о торможении + время, необходимое для перемещения ноги с педали газа на педаль тормоза, а также время отклика тормозной системы на нажатую педаль тормоза) составит где-то (50/10) х 3 = 15 метров. То есть пока вы будете принимать решение о торможении при скорости в 50 км/ч, ваша машина проедет 15 метров. 
  • Тормозной путь при нормальном торможении (с момента нажатия педали тормоза до момента остановки машины) составит около (50/10) х (50/10) = 25 метров. 
  • При экстренном торможении тормозной путь, как мы уже отметили, сокращается примерно в два раза. Соответственно, расчет тормозного расстояния автомобиля, который движется со скоростью 50 км/ч, будет выглядеть следующим образом: (50/10) x (50/10) / 2 = 12,5 метров.
  • В результате теперь мы можем вычислить реальный итоговый тормозной путь автомобиля. Так, при нормальном (не резком, а обычном) торможении итоговый тормозной путь составит около 40 метров. При экстренном торможении – не менее 28 метров. 

 

Примечание: Обратите внимание, что если скорость автомобиля будет выше всего в два раза, его итоговый тормозной путь увеличится в четыре раза!!!

 

Смотрите также: Основные принципы работы тормозного механизма автомобиля [Принцип работы и элементы тормозной системы]

 

То есть мнение о том, что при увеличении скорости автомобиля в два раза тормозной путь увеличивается только в два раза, – это чистый воды миф среди многих автолюбителей. Так что имейте это в виду, когда садитесь за руль. Самое удивительное, что об этом не знают даже многие опытные водители. 

 

Пример расчета тормозных и остановочных расстояний

Скорость, в км / ч

Путь, пройденный автомобилем

во время реакции водителя, в метрах

Тормозное расстояние, в метрах

(с момента нажатия педали тормоза

до полной остановки машины)

Итоговый тормозной путь, в метрах

25

7,5

6,25

13,75

50

15

25

40

100

30

100

130

150

45

225

265

200

60

400

460

 

Какие факторы влияют на торможение и тормозной путь?

 

Решающим значением для длины тормозного пути, конечно же, является скорость автомобиля, с которой он движется по дороге. Также на тормозной путь влияет качество установленной на машину тормозной системы. В том числе важную роль, несомненно, играет и состояние дороги (снег, лед, качество асфальта/бетона, трещины в дорожном покрытии, листья, лужи и т. п.). И само собой, не стоит забывать о состоянии шин автомобиля. Ведь в определенных случаях изношенная резина сильно увеличит тормозной путь автомобиля, так как не сможет передавать нормальную тормозную способность дорожному покрытию в отличие от новых шин, имеющих нормальное сцепление с дорогой. 

Также ясно, что на мокрой поверхности тормозное расстояние машины больше, чем на сухом асфальте. 

 

Не стоит забывать и об уровне подготовки водителя. Особенно важна, как мы узнали, для итогового тормозного пути скорость реакции водителя на дорожную ситуацию, требующую остановки автомобиля. Но скорость реакции за рулем зависит не только от опыта вождения. Например, знаете ли вы, что когда вы садитесь за руль в сонном состоянии (не выспались, устали или долго находились за рулем), то скорость реакции может замедлиться почти в два раза по сравнению со скоростью реакции хорошо отдохнувшего водителя. 

В целом же на скорость принятия решения за рулем (скорость реакции) влияет много факторов: возраст водителя, алкогольное или похмельное состояние, употребление определенных медикаментов и в целом состояние здоровья. Так, при многих хронических заболеваниях скорость реакции многих водителей существенно снижается. Следовательно, все эти факторы серьезно влияют на тормозной путь автомобиля. 

 

Смотрите также: Тормозной путь автомобиля: Все что нужно знать

 

То же самое касается и отвлечения внимания из-за смартфонов, которыми так любят пользоваться за рулем многие водители, несмотря на строгий запрет согласно нашему действующему законодательству.

 

Как мы уже сказали, на тормозной путь также влияет время отклика тормозной системы автомобиля на нажатую педаль тормоза. Особенно это касается старых автомобилей. Современные же, как правило, оснащены уже новым поколением тормозов, которые мгновенно активируются за счет максимального тормозного давления, как только вы резко ударите ногой по педали тормоза (например, при экстренном торможении). Эта технология позволила существенно сократить итоговый тормозной путь современных машин. 

 

Как повысить безопасность при управлении автомобилем?

 

Не зря основное правило вождения гласит о том, что водитель должен держать на дороге достаточную дистанцию до других автомобилей, чтобы оставалось пространство для экстренного торможения и для того, чтобы не спровоцировать ДТП. Но, с другой стороны, вы не должны держать дистанцию между автомобилями слишком большой. Помните, что все должно быть в меру. Вот некоторые правила вождения от экспертов:

 

  • В городском движении: Держите расстояние до других автомобилей около 15 метров. 
  • На автомагистралях, шоссе и проселочных дорогах: При скорости движения около 100 км/ч держите дистанцию примерно 50 метров. При плохой видимости или на скользкой дороге дистанция до других машин должна быть увеличена в два раза. Например, при скорости в 100 км/ч на скользкой дороге держите расстояние до впереди идущей машины минимум в 100 метров. 

1gai.ru

Как найти скорость торможения формула

Известно, что грузовой автомобиль массой пять тысяч килограмм движется по горизонтальному пути со скоростью семьдесят два километра в час (20 метров в секунду).
Необходимо: определить силу и время торможения автомобиля, если тормозной путь составил пять метров.

Дано: m=5000 кг; v=20 м/сек; s=5 м
Найти: F-?; t-?

Исходя из того, что работа силы торможения численно равна изменению кинетической энергии движущегося автомобиля , получаем формулу для определения силы торможения

Подставив в формулу численные значения, рассчитаем силу торможения грузового автомобиля

н

Из формулы , при условии, что vt=0: , где , получаем формулу времени торможения

Время торможения автомобиля

сек

Ответ: сила торможения автомобиля составила двести тысяч ньютон, время торможения равно половине секунды.

Тормозной путь – расстояние, которое потребуется автомобилю, чтобы полностью остановиться с момента начала работы системы торможения.

В обиходе этот термин часто путают с остановочным, однако тормозной и остановочный путь – разные понятия. В последнем случае учитывается расстояние, прошедшее с момента осознания водителем необходимости торможения до скорости 0 км/ч. Тормозной путь – часть остановочного.

От чего зависит тормозной путь

Рассматриваемый показатель не является постоянной величиной и может варьировать по ряду причин. Все факторы, влияющие на путь торможения, можно разделить на две большие группы: зависящие от водителя и независящие от водителя. К числу причин, не зависящих от человека за рулем, относят:

Несложно догадаться, что в дождь, снег или гололед расстояние, которое потребуется для остановки автомобиля, будет большим, чем на сухом асфальте. Торможение окажется длительным и при движении по гладкому асфальту, в который не была добавлена каменная крошка. Здесь колесам не за что зацепиться, в отличие от шершавых покрытий.

На заметку: стоит заметить, что плохое качество дороги (ямы, выбоины) не приводит к удлинению расстояния, необходимого для остановки. Здесь играет роль человеческий фактор. Пытаясь сберечь подвеску, водители редко развивают высокую скорость на подобных дорогах. Соответственно, путь торможения здесь минимален.

Факторы, зависящие от водителя или владельца авто:

  • состояние тормозов;
  • устройство системы;
  • наличие ABS;
  • вид покрышек;
  • загруженность ТС;
  • скорость движения.

Тот факт, что длина тормозного пути автомобиля напрямую зависит от исправности системы торможения, не требует доказательств. Машина с неработающим тормозным контуром или изношенными колодками никогда не сможет остановиться также быстро, как исправное ТС.

От устройства тормозных агрегатов зависит многое. Современные машины, оснащенные задними дисковыми тормозами и системами помощи при торможении, имеют гораздо лучшее сцепление с дорогой и короткий отрезок торможения.

В свою очередь, наличие EBD с ABS не всегда способствует сокращению расстояния, необходимого для остановки. На сухом твердом покрытии, где блокировка колес наступает только при очень интенсивном торможении, система действительно сокращает тормозной путь. Однако на голом льду «умный» электронный помощник начинает сбрасывать тормозное усилие даже при легком нажатии на педаль тормоза. При этом авто сохраняет управляемость, однако путь его торможения значительно увеличивается.

От чего зависит скорость замедления? Разумеется, от вида покрышек. Так, на голом, пусть и промороженном асфальте, а также в снежной каше, лучше всего тормозят т. н. «липучки» — зимние покрышки, не оснащенные шипами. В свою очередь, в гололед и на заснеженных дорогах наиболее эффективной является ошипованная «резина».
» alt=»»>
Немаловажным фактором, влияющим на величину остановочного отрезка, является скорость и загруженность машины.

Понятно, что легковесный автомобиль при скорости 60 км/ч остановится быстрее, чем грузовик, загруженный под завязку и движущийся со скоростью 80-100 км/ч. Последнему не позволит быстро остановиться слишком высокая для него скорость и инерция.

Когда и как производится замер

Расчет тормозного пути может потребоваться в следующих случаях:

  • технические испытания транспортного средства;
  • проверка возможностей машины после доработки тормозов;
  • криминалистическая экспертиза.

Как правило, при расчете используют формулу S=Кэ*V*V/(254*Фс). Здесь S – тормозной путь; Кэ – тормозной коэффициент; V₀ — скорость на момент начала торможения; Фс – коэффициент сцепления с покрытием.

Коэффициент сцепления с дорогой изменяется в зависимости от состояния покрытия и определяется по следующей таблице:

Состояние дорогиФс
Сухая0.7
Мокрая0.4
Снег0.2
Лед0.1

Коэффициент Кэ является статической величиной и составляет единицу для всех наиболее распространенных легковых транспортных средств.

Пример: как рассчитать тормозной путь автомобиля при цифре 60 км/ч на спидометре в дождь? Дано: скорость 60 км/ч, тормозной коэффициент – 1, коэффициент сцепления – 0.4. Считаем: 1*60*60/(254*0.4). В итоге получаем цифру 35.4, что и является длиной тормозного пути в метрах.

В таблице указано сколько метров машина будет продолжать движение до полной остановки. Следует учитывать, что в расчет не берутся никакие иные показатели (повороты, выбоины на дороге, встречный поток и т.д.). Сомнительно, что в реальных условиях на обледенелой дороге, автомобиль сможет проскользить километр и не встретить столб или отбойник.

СкоростьСухоДождьСнегЛед
км/чметры
6020,235,470,8141,7
7027,548,296,4192,9
8035,962,9125,9251,9
9045,579,7159,4318,8
10056,298,4196,8393,7
11068119238,1476,3
12080,9141,7283,4566,9
13095166,3332,6665,3
140110,2192,9385,8771,6
150126,5221,4442,9885,8
160143,9251,9503,91007,8
170162,5284,4568,81137,7
180182,2318,8637,71275,5
190203355,3710,61421,2
200224,9393,7787,41574,8

Мы нашли интересный калькулятор, который не только рассчитывает показатель в зависимости от скорости и состояния дороги, но и наглядно показывает весь процесс. Находится здесь.

Как увеличить интенсивность замедления

Из вышесказанного стало понятно, что называется тормозным путем и от чего зависит этот показатель. Однако возможно ли сократить расстояние, которое необходимо для остановки автомобиля? Возможно! Для этого существует два пути – поведенческий и технический. Идеально, если водитель сочетает оба способа.

  1. Поведенческий метод – сократить тормозной путь можно, если выбирать небольшую скорость движения на скользких и мокрых дорогах, учитывать степень загруженности машины, грамотно рассчитать тормозные возможности авто в зависимости от его состояния и модельного года. Так, «москвич» 1985 года разработки не сможет тормозить столь же эффективно, как современный «Hyundai Solaris», не говоря уж о более респектабельных и технологичных моделях.
  2. Технический метод – метод усиления тормозных возможностей, основанный на повышении мощности тормозной системы и использовании вспомогательных механизмов. Производители современных ТС активно применяют такие способы улучшения тормозов, оснащая свою продукцию антиблокировочными системами, системами помощи при торможении, используя более эффективные тормозные диски, колодки.

Следует помнить, что сокращение времени, необходимого для остановки – один из способов обеспечения безопасности поездки. Поэтому каждый водитель должен постоянно следить за техническим состоянием своего «железного коня», своевременно обслуживать и ремонтировать систему торможения. Помимо этого, важно выбирать скорость движения с учетом окружающей обстановки: времени суток, состояния дороги, модели автомобиля и прочее.

При торможении на автомобиль действует сила трения скольжения, поэтому по 2 закону Ньютона: µ mg = ma , отсюда а = µg. Путь до полной остановки рассчитывается по формуле: S = v 2 / 2 a . Тогда тормозной путь равен : S = v 2 / 2µ g . Остановочный путь равен сумме пути автомобиля во время реакции водителя (равномерное движение) и тормозного пути: S = vt + ( v 2 / 2µ g ) .

При повороте на тело также действует сила трения, но тело движется по окружности, поэтому 2 закон Ньютона будет иметь вид: µ mg = mv 2 / R . Тогда радиус поворота равен: R = v 2 / µ g .

Не вписался в поворот

1. Вычислить остановочный путь автомобиля для начальной его скорости 72 км/ч , если он замедляется с постоянным ускорением 6 м/с 2 , а время реакции водителя составляет 1с.

2. Шофер автомобиля, едущего со скоростью 60 км/ч, внезапно видит перед собой группой школьников, собирающейся перейти дорогу. Как поступить целесообразнее: затормозить или повернуть?

Для домашнего задания:

1. На трассе за чертой города скорость автомобиля 110 км/ч. Включен дальний свет. Дорогу перебегает заяц. Какова его судьба?

2. На горизонтальной дороге автомобиль делает поворот радиусом 16 м. Какова наибольшая скорость, которую может развить автомобиль, чтобы его не занесло, если коэффициент трения скольжения колес о дорогу равен 0,4?

3. Шофер автомобиля, едущего со скоростью 60 км/ч, внезапно видит перед собой на расстоянии 40 м широкую стену. Что ему выгоднее: затормозить или повернуть?

4. Выясните технические характеристики вашего семейного автомобиля (если в семье нет автомобиля – выберите в интернете понравившуюся модель и изучите ее). Какие технические средства безопасности пассажиров есть в автомобиле, выясните наличие антиблокировочная тормозная система и узнайте мнение водителя о ее роли.

5. При возможности практически выясните глубину зоны видимости при ближнем свете, входя в эту зону в темной одежде и со светоотражателем – например со знаком аварийной остановки в руках. Автомобиль стоит.

l2rv.ru

Тормозной путь, формула — Лада мастер

Беспечность выглядит эффектно только в хорошо продуманных сценах из боевиков и детективов. На самом же деле, большинство водителей даже не представляют, о какой опасности идёт речь, когда говорят о соблюдении дистанции и о превышении скорости. Многие ли падали с трехметровой высоты плашмя на бетонный пол? Едва ли. А на самом деле, точно такую же нагрузку будет испытывать человек в автомобиле при наезде на неподвижное препятствие на скорости… всего 28 км/ч.

Содержание:

Зачем знать длину тормозного пути

Раз уж мы начали с расчётов, говоря о длине тормозного пути движущегося автомобиля, используем простую физическую формулу, известную каждому школьнику. Её используют для вычисления перехода энергии падения в кинетическую энергию конце пути (mgh=mVx2/2). Отсюда получаем, что при скорости около 30 км/ч тело получает удар, равный падению с высоты три метра. Соответственно, при движении на скорости 60 км/ч сила удара будет равна падению с высоты 15м, а уже на скорости 90 км/ч — с высоты около 32 м, 120 км/ч — это уже высота 55 метров.

Даже учитывая, что в автомобиле срабатывает подушка безопасности, выжить при лобовом ударе на скорости 60 км/ч шансов очень мало. Это примерная высота хрущевки. Отважится ли кто-то прыгнуть с крыши пятиэтажки, обвязавшись надувными подушками? Едва ли. А что говорить о скорости в 90 км/ч, удар при которой равносилен падению с высоты десятиэтажного дома? А с высоты 55 метров? Шансов выжить никаких, и это даже при условии, что подушка безопасности сработает безукоризненно.

Эмпирическая формула расчёта тормозного пути

Имея отличный водительский глазомер и достаточный опыт, каждый сможет определить расстояние до объекта на глаз, хотя бы примерно. Водительский опыт показывает, что для мгновенного вычисления длины тормозного пути по скорости, необходимо просто бросить взгляд на спидометр, оценить расстояние до препятствия, тогда тормозной путь будет равен половине числа, которое показывает спидометр. То есть, исходя из эмпирической формулы расчёта длины тормозного пути, безопасная дистанция до любого объекта будет равна мгновенной скорости, разделённой пополам. Практически так же производят расчёт скорости автомобиля по тормозному пути.

При этом нужно учитывать такое понятие, как остановочный путь, это термин экспертов дорожной полиции и он учитывает не только сам по себе тормозной путь, но и скорость реакции, а также время реагирования системы тормозов. В принципе — это расстояние до абсолютной остановки машины от того момента, когда водитель зафиксировал препятствие. Естественно, остановочный путь всегда больше тормозного, поскольку средняя скорость реакции здорового и трезвого водителя около 0,8 с, а тормозная система срабатывает ещё за 0,2-0,3 с. Следовательно, до полной остановки машины пройдёт ещё 1,1 с, а на скорости 60 км/ч автомобиль проходит 16,6 метров за одну секунду. Почти семнадцать метров, которые неминуемо будут добавляться к длине тормозного пути и которые редко учитываются большинством водителей. Вот именно поэтому необходимо серьёзно отнестись хотя бы к теоретическому вычислению длины тормозного пути.

Что нужно для расчёта тормозного пути

Чтобы вычислить тормозной путь формула которого указана на рисунке с пояснениями, мало знать моментальные сухие данные.

Теоретически, для оценки тормозных характеристик машины необходимо использовать массу данных:

  • длину тормозного пути;
  • минимальное время, за которое тормозная система сработает;
  • диапазон изменения тoрмозных усилий;
  • алгоритм изменения тoрмозных усилий;
  • производительность тормозов в зависимoсти от нагрева;
  • качество дорожного покрытия;
  • эффективность подвески автомобиля;
  • степень износа и тип покрышек.

Здесь нужно учитывать целый ряд моментов. К примеру, эффективность работы тормозной системы в каждом автомобиле может быть разной и это само собой разумеется. Гидравлическая система тормозов даёт задержку минимум 0,2-03 с, а пневматика, установленная на большинстве грузовиков и автобусов и того больше, до 0,6 с. Кроме этого, есть такое понятие, как нарастание тормозного усилия с нуля до максимального значения и это также отбирает от 0,4 до 0,6 с, при этом влияние скорости движения на длину тормозного пути в этом случае увеличивается в квадрате, то есть при увеличении скорости в два раза, тормозной путь будет вчетверо длиннее.

Дополнительные составляющие тормозного пути

При вычислении эффективности тормозов очень большое значение имеет характеристика подвески и состояние шин. При чем тут подвеска? Очень просто. У нас под колёсами довольно редко встречается идеально ровный асфальт, а именно подвеска, точнее, амортизаторы, рессоры, торсионы и пружины как раз и прижимают колеса к поверхности, делая торможение и управление максимально эффективным. Если амортизатор неисправен, колеса подпрыгивают на ухабах и о полном контакте с покрытием не может быть и речи.

Давайте к этому прибавим кoэффициент сцепления резины с дорoгой — здесь огромное значение имеет состояние дороги, тип покрышки (зима  или лето), рисунок протектора, геометрия, износ прoтектора и качество резиноматериала. Тесты показали, что на одном и том же автомобиле, но с разными покрышками, длина тормозного пути может изменяться до трёх-пяти метров, а о качестве пoкрытия и говорить нечего. Попробуйте сравнить тoрможение на сухом асфальте и на льду.

Как видим, факторов, влияющих на тормозной путь, а тем более на остановочный, достаточно много, поэтому предельная концентрация внимания за рулём — это гарантия безопасной езды. Проверяйте тормоза вовремя, не говорите по телефону за рулём и пусть все ваши дороги будут добрыми!

ladamaster.com

Как найти путь торможения физика

Тормозной путь — расстояние, которое проходит транспортное средство с момента срабатывания тормозной системы до полной остановки.

Протяжённость тормозного пути зависит от скорости, состояния проезжей части, шин, погодных условий. Особое влияние на протяжённость тормозного пути оказывает эффективность тормозной системы (ТС). Она складывается из технологических особенностей узлов ТС — «Электронных помощников», логики их работы, диаметра тормозных дисков, материала тормозных колодок, принудительной вентиляции и других параметров.

А) модуль силы трения, действующей

Б) тормозной путь грузовика

А) Сила трения при торможении грузовика равна . Это соответствует варианту ответа 1.

Б) Грузовик тормозит за счет силы трения, то есть с ускорением

Время торможения до полной остановки можно найти как

.

Тогда тормозной путь будет равен

и подставляя вместо t и a найденные выражения, получаем:

.

Калькулятор тормозного пути позволит оценить тормозной путь автомобиля, движущегося с заданной скоростью. Для использования укажите тип дорожного покрытия, на котором тормозит автомобиль и скорость, при которой начинается торможение. Калькулятор рассчитает сколько метров пройдет автомобиль при торможении.

Калькулятор тормозного пути

Формула тормозного пути

Формула для нахождения тормозного пути

Формула для нахождения тормозного пути применяется в подразделениях ГИБДД. Именно она используется в нашем калькуляторе. В этой формуле:

S — тормозной путь,

Кт — тормозной коэффициент (для легкового автомобиля равен 1),

V — скорость автомобиля,

Kсц — коэффициент сцепления.

Понятия и пояснения

Тормозной путь — это путь, который проходит автомобиль с момента, когда сработал тормозной механизм до полной остановки автомобиля. На него влияют:

  • состояние и тип дорожного покрытия,
  • состояние шин автомобиля,
  • начальная скорость автомобиля,
  • масса автомобиля,
  • исправность тормозной системы.

Остановочный путь — путь с момента обнаружения опасности до полной остановки автомобиля. Понятно, что тормозной путь входит в остановочный. Кроме того в остановочный путь входят:

  1. путь, который проехал автомобиль с момента обнаружения опасности до нажатия на педаль тормоза;
  2. путь, пройденный автомобилем за время срабатывания тормозной системы.

Первый параметр зависит от множества факторов, определяющим из которых является времени реакции водителя. По результатам многочисленных экспериментов, оно может меняться от 0,3 до 1,5 секунды. В среднем можно считать время реакции водителя равное 1 секунде. Кроме этого существует понятие «нормативное время восприятия сложной ситуации» равное 0,8 секунды. Также установлено, что время реакции у женщин, при возникновении сложной дорожной ситуации может достигать 2,5-3 секунд, тогда как у мужчин 1,5-2 секунды. Кроме этого на время реакции влияет:

  • опыт водителя,
  • его эмоциональное состояние,
  • возраст,
  • время суток и погодные условия,
  • прием медикаментов,
  • состояние алкогольного или иного опьянения,
  • место возникновения опасной ситуации.

Время срабатывания тормозной системы зависит от ее типа и технического состояния. Тормозная система с гидравлическим приводом срабатывает за 0,2 – 0,3 секунды, с пневматическим за 0,5 –0,6 секунд.

l2rv.ru

Как определить тормозной путь формула физика

Сухой асфальт

На сухом асфальте коэффициент сцепления шины составляет 0,7–0,8. Это отличный показатель.

Мокрый асфальт, лёд, снег

На мокром асфальте коэффициент сцепления составляет 0,4–0,5.

Виды торможения

Сначала рассмотрим способы:

  • газ-тормоз;
  • ступенчатый, с понижением усилия;
  • ступенчатый, с повышением усилия;
  • прерывистый;
  • силовое торможение мотором;
  • торможение силовым агрегатом.

А теперь рассмотрим виды:

  1. Аварийное. Аварийное торможение используется тогда, когда обычные способы не приносят необходимых результатов.
  2. Стояночное. Для торможения применяется ручной тормоз. Стояночное торможение применяется для фиксации транспортного средства в состоянии покоя.
  3. Экстренное. Такой способ используется при возникновении экстренной ситуации. Такой способ позволяет максимально быстро замедлить машину.
  4. Служебное. Это стандартный способ. Существует два варианта:
  • до полной остановки машины — применяется для полной остановки машины;
  • частичное — этот способ используется для снижения скорости.

Экстренное торможение

Как правило, экстренное торможение используется в старых машинах, которые не оборудованы современными системами безопасности (ESP, ABS и т. д.).

Определение скорости авто с помощью тормозного пути

Проводить расчёт по формуле достаточно сложно. Для определения скорости машины можете воспользоваться специальными онлайн-калькуляторами. Найти такой онлайн-калькулятор можно в поисковой системе.

Онлайн-калькуляторы разработаны с учётом всех требований. В них учтены все данные и формулы.

Вам нужно только вести такие данные:

  • длина следа торможения;
  • вид дорожного покрытия;
  • степень загрузки транспортного средства;
  • тип автомобиля;
  • скорость движения.

Далее, всю работу за вас сделает онлайн-калькулятор.

А теперь рассмотрим формулу для определения скорости движения. Формула: 0.5 х t3 х j + √2Sю х j.

Описание:

  • Sю — это длина следа;
  • j — этот символ обозначает замедление транспортного средства при торможении;
  • t3 — это нарастание замедления машины;
  • Va — начальная скорость машины.

Отличие тормозного пути от остановочного

Рассмотрим подробнее эти два понятия.

  1. Тормозной. Это расстояние, проходимое машиной с момента нажатия на педаль. Он является частью остановочного пути.
  2. Остановочный. Это расстояние, проходимое машиной с момента обнаружения опасности.

Определить тормозной путь автомобиля можно двумя способами:

  • провести расчёты по специальной формуле;
  • воспользоваться онлайн-калькуляторами, которые можно найти на профильных сайтах.

На тормозной путь влияет большое количество факторов (дорожное покрытие, состояние транспортного средства, погодные условия, навыки водителя, способ торможения, протектор шин).

Не нашли ответа на свой вопрос? Узнайте, как решить именно Вашу проблему — позвоните прямо сейчас:

+7 (499) 455-03-75 (Москва)
+7 (812) 407-26-30 (Санкт-Петербург)

Может случиться так, что от длины тормозного пути будет зависеть целостность кузова автомобиля и сохранность его пассажиров. Автомобиль на скорости просто не может резко замереть после нажатия на тормоз, даже если на нем стоят качественные покрышки и эффективная система торможения. После того, как нажата педаль тормоза, машина в любом случае преодолевает некоторое расстояние, и называется это расстояние — тормозной путь.

Водителю необходимо постоянно рассчитывать длину тормозного пути в соответствии с одним из правил по безопасности движения, которое говорит о том, что путь торможения должен быть меньше, чем расстояние до помехи.

В данной ситуации, все зависит от реакции и умения водителя, чем раньше он нажмет на тормоз и правильнее рассчитает длину тормозного пути, тем раньше, и успешнее авто затормозит.

Тормозной путь автомобиля при скорости 60 км/ч

Длина остановочного пути также зависит не только от водителя, но и от других сопутствующих факторов: от качества дороги, скорости движения, погодных условий, состояния тормозной системы, устройства тормозной системы, шин автомобиля и многих других.

Обратите внимание, что вес легкового автомобиля не влияет на длину тормозного пути. Это связано с тем, что вес автомобиля увеличивает инертность автомобиля при выполнении торможения, препятствуя при этом торможению, но увеличивает сцепление шин с дорогой благодаря увеличенной массе авто.

Эти физические свойства компенсируют друг друга, при этом практически не оказывая влияние на длину тормозного пути.

Скорость торможения напрямую зависит от способа торможения. Резкий тормоз до упора, приведет к заносу или движению машины юзом (если машина не оборудована системой ABS).

Постепенное нажатие на педаль применяется когда на дороге хорошая видимость и спокойная обстановка, оно не подходит для экстренных ситуаций. При прерывистом нажатии можно потерять управляемость, но зато быстро остановиться. Также возможно ступенчатое нажатие (схоже по эффекту с системой АБС).

Существуют специальные формулы, которые позволяют определить длину тормозного пути. Мы попробуем просчитать формулу по разным условиям, в зависимости от типа дорожного покрытия.

Формула для определения тормозного пути
Тормозной путь на сухом асфальте

Вспоминаем уроки физики, где ? – это коэффициент трения, g – ускорение свободного падения, а v – скорость движения машины в метрах в секунду.

Ситуация следующая: едет водитель на автомобиле Lada скорость которого 60 км/час. Буквально в 70 метрах идет женщина преклонного возраста, которая забыв о правилах безопасности спешно догоняет маршрутное такси (стандартная ситуация для России).

Воспользуемся этой самой формулой: 60 км/ч = 16,7 м/сек. У сухого асфальта коэффициент трения равняется 0,7 , g – 9,8 м/с. На самом деле, в зависимости от состава асфальта, он равен от 0.5 до 0.8, но всё же возьмем усредненное значение.

Полученный по формуле результат 20,25 метров. Естественно, что данное значение уместно лишь для идеальных условий, когда на машину установлена качественная резина и тормозные колодки, тормозная система исправна, при торможении вы не уходите в юз и не теряете управление, от множества других идеализированных факторов, которые не встречаются в природе.

Также для перепроверки результата, существует еще одна формула определения тормозного пути:

S = Кэ * V * V / (254 * Фс) , где Кэ – тормозной коэффициент, для легковых авто он равняется единице; Фс – коэффициент сцепления с покрытием 0,7 (для асфальта).

Подставляем скорость движения транспортного средства в км/ч.

Получается, что тормозной путь 20 метров для скорости 60 км/ч, (для идеальных условий), в том случае если торможение будет резким и без юза.

Тормозной путь на покрытии: снег, лед, мокрый асфальт

Коэффициент сцепления помогает обозначить длину остановочного пути при разных дорожных условиях. Коэффициенты для разных дорожных покрытий:

  • Сухой асфальт – 0,7
  • Мокрый асфальт – 0,4
  • Укатанный снег – 0,2

Попробуем подставить эти значения в формулы, и найдем значения длины тормозного пути для дорожного покрытия в разное время года и при разных погодных условиях:

  • Мокрый асфальт – 35,4 метра
  • Укатанный снег – 70,8 метра
  • Лед – 141,6 метра

Получается, что на льду длина тормозного пути практически в семь раз выше, относительно сухого асфальта (так же как и подставляемый коэффициент). На длину тормозного пути влияет качество зимней резины, физические свойства.

Тестирование показало, что с системой АБС остановочный путь существенно снижается, но все же при гололеде и снеге АБС не влияет, а наоборот ухудшает эффективность торможения, если ее сравнивать с тормозной системой без ABS. Тем не менее, в АБС по большей мере все зависит от настроек и наличия системы распределения тормозного усилия (ЕБД).

Преимущество АБС в зимнее время – полный контроль над управлением автомобиля, что сводит к минимуму возникновения неуправляемого заноса при выполнении торможения. Принцип работы АБС схож с выполнением ступенчатого торможения на автомобилях без АБС.

Система АБС уменьшает тормозной путь на: сухом и мокром асфальте, укатанном гравии, разметке .

На льду и укатанном снеге использование АБС увеличивает тормозной путь на 15 — 30 метров, но позволяет сохранить контроль над машиной, без увода машины в занос. Этот факт следует учитывать.

Как тормозить на мотоцикле?

Правильно тормозить на мотоцикле задача довольно сложная. Можно тормозить задним колесом, передним, либо двумя, юзом или двигателем. При неправильном торможении на больших скоростях можно потерять равновесие. Для того, чтобы рассчитать тормозной путь мотоцикла на 60 км/ч также подставляют данные в формулу. Учитывая при этом другой тормозной коэффициент и коэффициент трения.

Тормозной путь мотоциклов

  • Сухой асфальт: 23 — 33 метра
  • Мокрый асфальт: 35 — 46 метра
  • Грязь и снег: 70 — 95 метра
  • Гололед: 95 — 128 метра

Второй показатель – тормозной путь при торможении мотоцикла юзом.

Длину тормозного пути должен знать и уметь рассчитать любой владелец транспортного средства, и лучше это делать визуально.

Следует помнить, что при возникновении дорожно-транспортного происшествия по длине юза, который останется на дорожном покрытии, можно определить скорость движения транспортного средства перед столкновением с препятствием, что может констатировать превышение допустимой скорости водителем и сделать из него виновника происшествия.

Прием заказов на покупку автомобиля стартовал в марте, и уже на днях автомобиль поступил в продажу. Новинку отличает переработанный задний бампер с центральной подножкой, рулевое колесо и рычаг коробки передач с кожаной отделкой.

На индийском рынке появится доработанный кроссовер Suzuki Vitara Brezza. Автомобиль выйдет под брендом Toyota в 2019 году. Модель пользуется большим спросом на местном рынке. В общей сложности, было продано 210 тысяч единиц.

  • 10 — 11 классы
  • Физика
  • 5 баллов

Определить тормозной путь автомобиля если он двигался со скоростью 72км/ч и остановился за 5 с

«>

litezona.ru

что это, значение, принцип работы

Тормозной путь автомобиля — это дистанция до полной остановки, которую успевает преодолеть машина с того момента, как водитель нажал педаль тормоза. Важно понимать, что остановочный путь всегда больше, чем тормозной. Ведь он включает еще и расстояние, пройденное с того момента, как водитель обнаружил опасность и нажал на тормоза.

Как рассчитать тормозной путь

Длина пути рассчитывается по следующей формуле:

l = V2/(2µg), где

  • l — путь,пройденный автомобилем;

  • V — скорость авто;

  • µ — коэффициент, определяющий силу трения

  • g — ускорение свободного падения (9,8м/с2).

Скорость легко определяется по показаниям спидометра, а коэффициент трения покрышек на сухом асфальте колеблется в диапазоне 0,5-0,8.

Для приблизительных расчетов используется µ=0,7.

Для скорости 60 км/час (по системе СИ — 16,7 м/с) тормозной путь равен:

16,72/(2*0,7*9,8)=20,24 метров.

Столько проедет серийная машина с момента начала торможения.

Однако такое значение актуально лишь для условий, приближенных к идеальным. При неравномерном срабатывании тормозов (цилиндров и колодок на каждом колесе) машина может потерять управляемость. Для восстановления контроля придется ослабить нажатие на тормоз. В этом случае тормозной путь будет значительно длиннее.

При расчете пройденного расстояния учитывается квадрат скорости. То есть, с ростом скорости тормозной путь резко удлиняется. При 80 км/час он составит уже 36 м, а на 120 км/час — 81 метр.

От чего зависит длина тормозного пути

Как видим, на тормозной путь влияют два параметра: скорость и коэффициент трения. Если скорость полностью зависит от действий водителя, то с трением все значительно сложнее. Давайте разберемся, какие факторы на расстояние, необходимое машине для остановки.

Состояние шин

Коэффициент сцепления (µ) зависит от следующих параметров:

Изношенный протектор сильно ухудшает торможение на мокрой, заснеженной или даже грязной дороге. Зато зависимость тормозного пути от температуры нелинейна.

При низких температурах резина теряет эластичность и коэффициент трения уменьшается. Поэтому в холодное время года нужно использовать зимние шины независимо от того, как успешно дорожные службы справляются с уборкой снега. Зимой даже на чистом асфальте тормозной путь на зимней резине будет намного короче, чем на летней.

При высоких температурах резина становится слишком мягкой. При этом она интенсивно изнашивается и начинает легче скользит по асфальту. Поэтому летом быстрее остановиться получится на летней резине, которая сохраняет эластичность, но не «течет» подобно пластилину.

Дорожное покрытие

Коэффициент трения, который на сухом асфальте равен 0.7, меняется в зависимости от погодных условий:

  • 0,1 — гололед;

  • 0,2 — снежный накат;

  • 0,4 — мокрый асфальт.

В летнее время нужно остерегаться больших луж и грязи. Лужи могут вызвать эффект аквапланирования, при котором сцепление с дорогой будет даже хуже, чем на укатанном снегу. Не менее опасна и грязь: тонкий слой мокрой глины, практически невидимый глазом, делает асфальт таким скользким, что на нем становится сложно просто устоять на ногах.

Антиблокировочная система

Как известно из школьного курса физики, сила трения скольжения всегда ниже, чем трения покоя. То есть, при торможении «юзом» тормозной путь больше. Этот эффект давно известен опытным водителями. Чтобы быстрее остановиться и не потерять управление на скользкой дороге, они используют «прерывистое» торможение. Метод заключается в том, что при блокировке колес водитель на мгновение отпускает педаль тормоза и тут же нажимает ее снова.

На большинстве современных серийных авто устанавливается электронная антиблокировочная система. Она контролирует вращение каждого колеса и снижает давление в тормозной магистрали при блокировке. В отличие от «прерывистого торможения», ABS контролирует каждый тормозной цилиндр в отдельности и ослабляет торможение только для заблокированных колес. За счет этого удается достичь минимального тормозного пути на сухом асфальте, гололеде и мокрой дороге.

Однако антиблокировочная система не всегда позволяет остановить авто быстрее, чем торможение «юзом». На снегу и грязи она не позволяет протектору поглубже «зарыться» в дорогу. Особенно заметен эффект при использовании шипованной резины. Поэтому если вы хотите, чтобы шипы эффективно тормозили, «вгрызаясь» в снег, лед или грязь, ABS стоит отключить.

Тормозной путь автомобиля: определение, формула, расчет

Автоликбез14 сентября 2019

Каждому водителю важно помнить, что его машина не может остановиться мгновенно. Для этого ему потребуется определенное время, на которое влияет большое количество факторов. Правила дорожного движения требуют соблюдать безопасное расстояние между собственным и впереди идущим автомобилем, чтобы в случае необходимости успеть затормозить. Чтобы знать величину этого расстояния, необходимо иметь представление о тормозном пути. Помимо этого, многие путают два понятия – тормозной и остановочный путь.

Понятие тормозного пути автомобиля

Даже, если за рулем машины сидит профессиональный водитель, на дороге всегда может возникнуть ситуация, когда необходимо максимально быстро остановить транспортное средство:

  • внезапное появление на дороге человека или животного;
  • неисправность транспортного средства;
  • нарушение другим водителем правил дорожного движения, что приводит к созданию аварийной ситуации;
  • непредвиденные обстоятельства: неровность дорожного покрытия, препятствие (упавшее дерево, камень) и т.п.

Для остановки автомобиля водитель использует педаль тормоза, приводя в работу его тормозную систему.

Тормозной путь авто – это расстояние, которое преодолевает транспортное средство за период времени с момента срабатывания системы торможения до достижения транспортным средством скорости 0 км/ч.

От чего зависит тормозной путь?

Очевидно, что дистанция торможения будет различной в зависимости от ситуации и ее условий. Так, факторы, влияющие на величину этого пути, делят на две группы:

  1. Факторы, которые зависят от автомобилиста.
  2. Факторы, которые не зависят от автомобилиста.

К условиям, которые не зависят от того, кто управляет автомобилем, относят погоду и состояние дорожного покрытия. Что касается погоды, то логично, что в дождь, снег или гололед времени для остановки машины потребуется больше, чем в сухую погоду.

Дорожное покрытие тоже оказывает влияние на расстояние торможения. Если дорога гладкая без добавления камня, то дистанция, которая будет пройдена транспортным средством при торможении, также будет больше.

На заметку! Если на дорогах есть ямы, то, скорее всего, тормозной путь будет коротким. Это связано с тем, что на таком плохом участке дороге автомобилист просто не будет развивать высокую скорость.

Гораздо больше факторов, которые зависят от водителя (владельца машины):

  • скорость. Логично, чем меньше скорость, тем короче расстояние торможения;
  • состояние и устройство тормозной системы. Важно, чтобы машина, в том числе ее тормоза, работала исправно, чтобы колодки не были изношены, а давление в шинах было достаточным.
  • вид установленных шин. Протектор не должен быть сильно изношен, а тип установленной резины должен соответствовать погодным условиям;
  • загрузка автомобиля. Чем легче транспортное средство, тем проще его остановить. Расстояние торможения нагруженного автомобиля будет более длинным;
  • наличие системы ABS. На сухом асфальте данная система поможет остановить машину быстрее, а вот в гололед она позволит сохранить управление, но дистанция торможения при этом станет длиннее;
  • трезвое состояние водителя. Адекватный водитель быстрее реагирует на быстро меняющуюся ситуацию на дороге, благодаря чему, он быстрее остановит свой транспорт при необходимости;
  • отсутствие отвлекающих факторов во время движения. Зачастую замедленная реакция автомобилиста связана с тем, что он отвлекается и не следит за дорогой. Самый распространенный фактор отвлечения внимания – это мобильный телефон. Из-за замедления реакции того, кто управляет авто, путь торможения увеличивается.

Формула расчета тормозного пути

Иногда необходимо рассчитать величину тормозного пути, например в таких случаях:

  • испытания автомобиля;
  • криминалистическая экспертиза;
  • проверка работы тормозной системы авто после ее доработки.

Для выполнения такого расчета используют следующую формулу:

Sторм = Кэ * V * V / (254 * Фс), где:

Sторм – путь торможения;

Кэ – коэффициент торможения;

V – скорость машины;

Фс – коэффициент сцепления.

Последний коэффициент может быть разным. Так:

  • при сухой дороге он равен 0,7;
  • при мокрой – 0,4;
  • при снеге – 0,2;
  • при гололеде – 0,1.

Что касается коэффициента торможения, то он является постоянной величиной и чаще всего равняется единице.

Приведем пример. Машина движется летом по сухому асфальту со скоростью 80 км/ч. Необходимо рассчитать величину пути торможения.

S = 1 * 80 * 80 / (254 * 0,7) = 36 метров – это и есть расстояние торможения.

Важно знать! Тормозная дистанция авто прямо пропорциональна квадрату его скорости. Таким образом, увеличивая скорость в два раза, например, с 40 км/ч до 80 км/ч, расстояние торможения увеличивается в четыре раза.

Чем отличается тормозной путь от остановочного?

Тормозной и остановочный пути – это разные понятия, которые часто путают или принимают за одно и тоже.

Остановочный путь – это расстояние, которое прошло транспортное средство с момента осознания автомобилистом необходимости в остановки до достижения машиной скорости 0 км/ч.

А тормозной путь – это дистанция, которую прошла машина с момента срабатывания ее тормозов до ее остановки.

Таким образом, остановочное расстояние включает в себя не только дистанцию торможения, но и расстояние, которое прошло транспортное средство, пока автомобилист реагировал на дорожную ситуацию.

Как рассчитать полное время остановки и итоговый тормозной путь?

Итак, итоговое значение этого пути включает в себя не только расстояние торможения, но и дистанцию реакции автомобилиста.

Чтобы рассчитать расстояние, которое пройдет авто за время реакции водителя, необходимо воспользоваться следующей формулой:

Sреакции = V / 10 * 3, где

V – это скорость транспортного средства.

Таким образом, итоговый тормозной путь будет равняться сумме двух значений: пути реакции автомобилиста и пути торможения:

Sитог = Sторм + Sреакции

Возвращаясь к примеру, в котором машина движется летом по сухому асфальту со скоростью 80 км/ч, рассчитаем дистанцию реакции.

Sреакции = 80/10 * 3 = 24 метра

Теперь, когда мы знаем, что дистанция торможения равна 36 метрам, а расстояние реакции – 24 метра, можно рассчитать его итоговое значение:

Sитог = 36 + 24 = 60 метров

Соответственно, полное время остановки – это временной период, за который машина пройдет итоговый тормозной путь. Это время складывается из времени реакции водители и времени, затраченного на тормозную дистанцию.

Формула его расчета следующая:

, где:

– время реакции водителя;

– время срабатывания тормозного привода;

– время нарастания тормозных сил;

– начальная скорость торможения;

– ускорение свободного падения;

– коэффициент продольного сцепления с дорогой колёс автомобиля;

– коэффициент эффективности торможения.

Важно! Общепринятая норма времени реакции автомобилиста равняется одной секунде.

Итак, итоговое остановочное расстояние включает в себя дистанцию реакции водителя и тормозной путь. На каждую из этих величин влияют определенные факторы. Чтобы сократить значение итоговой величины, необходимо соблюдать скоростной режим, следить за исправностью автомобиля, учитывать его загруженность и садиться за руль исключительно в адекватном состоянии.

Тормозной путь

Тормозной путь — это расстояние, которое автомобиль проезжает при замедление до полной остановки. Тормозной путь зависит от нескольких переменные. Во-первых, на торможение влияет уклон (уклон) проезжей части. расстояние. Если вы идете в гору, сила тяжести помогает вам в попытках остановиться и уменьшает тормозной путь. Точно так же гравитация работает против вас, когда вы при спуске и увеличит тормозной путь.Далее сопротивление трения расстояние между проезжей частью и шинами может повлиять на тормозной путь. Если у тебя есть старые шины на мокрой дороге, скорее всего, вам потребуется большее расстояние, чтобы остановиться, чем если бы у вас новая резина на сухой дороге. Последний параметр, который мы рассмотрим, — это ваш начальный скорость. Очевидно, что чем выше ваша скорость, тем дольше вы будете останавливаться, учитывая постоянное замедление.

Уравнение, используемое для расчета тормозного пути, является дочерним по отношению к более общим уравнение из классической механики.Исходное уравнение приведено ниже.

Vf2 = Vo2 + 2ad

Где:
Vf = Конечная скорость
Vo = начальная скорость
a = Скорость ускорения
d = расстояние, пройденное при ускорении

При расчете тормозного пути мы предполагаем, что конечная скорость будет равна нуль. Исходя из этого, уравнением можно манипулировать, чтобы найти расстояние пройдено при торможении.

d = -Vo2 / (2a)

Обратите внимание, что расстояние будет положительным, пока отрицательная скорость ускорения использовал.

Ускорение тормозящего транспортного средства зависит от сопротивления трения и класс дороги. Из наших знаний о силе трения мы знаем, что ускорение из-за трения можно рассчитать, умножив коэффициент трения ускорением свободного падения. Точно так же мы знаем из задач наклонной плоскости что часть веса автомобиля будет действовать в направлении, параллельном поверхности Дорога.Ускорение свободного падения, умноженное на уклон дороги, даст нам оценить ускорение, вызванное уклоном дороги.

Окончательная формула тормозного пути приведена ниже. Обратите внимание, как Скорость ускорения рассчитывается путем умножения ускорения свободного падения на сумму коэффициента трения и уклона дороги.

d = V2 / (2g (f + G))

Где:
d = тормозной путь (фут)
g = ускорение свободного падения (32.2 фут / сек2)
G = уклон проезжей части в процентах; для 2% используйте 0,02
V = Начальная скорость автомобиля (фут / сек)
f = коэффициент трения между шинами и дорожным полотном

Тормозной путь и время реакции тормоза являются важными составляющими расчет дальности остановки прицела. Для того, чтобы стопорный прицел при условии достаточного расстояния, нам необходимо более глубокое понимание фрикционных сила.Значение коэффициента трения сложно определить. определять. Сила трения между шинами и дорожным полотном сильно различается. и зависит от давления в шинах, состава шин и типа протектора. Фрикционный сила также зависит от состояния поверхности дорожного покрытия. Наличие влага, грязь, снег или лед могут значительно уменьшить тормозящую силу трения ты. Кроме того, коэффициент трения ниже на более высоких скоростях.С коэффициент трения для мокрого покрытия ниже, чем коэффициент трения для сухое покрытие, мокрое покрытие используется на расстоянии видимости остановки расчеты. Это обеспечивает разумный запас прочности, независимо от состояние дорожного покрытия. В таблице ниже приведены несколько значений фрикционного коэффициент в условиях мокрого дорожного покрытия (ААШТО, 1984).

Понимание остановки и тормозного пути в физике — видео и стенограмма урока

Этот урок исследует физику расстояния, необходимого для остановки движущегося автомобиля.Вы узнаете разницу между дистанцией мышления, тормозным путем и тормозным путем. Наконец, мы рассчитаем, как далеко автомобиль остановится, исходя из его начальных условий и как оценить это расстояние.

Торможение и остановка

Представьте себе автомобиль, движущийся со скоростью 73 км / час (примерно 45 миль / час), когда цвет светофора на перекрестке меняет цвет с зеленого на желтый. Водителю предстоит принять решение — проехать на желтый свет или остановиться на белой линии, разделяющей перекресток.Давайте подробно рассмотрим процесс торможения и остановки, начиная с момента, когда светофор становится желтым.

В нашем сценарии водитель решает остановиться на перекрестке. Желтые огни, вообще говоря, не вызывают инстинктивной реакции водителей на торможение. Если водитель находится прямо на перекрестке, они обычно проезжают его. Некоторые люди проезжают перекресток даже после того, как загорелся красный свет, что является незаконным и опасным! Наш водитель находился на таком расстоянии от перекрестка, где он мог бы безопасно остановиться или безопасно проехать перекресток, если бы он увеличил скорость.

Принятие решения об остановке требует времени, не много времени, но это действительно требует времени. Расстояние, которое проезжает автомобиль, когда мозг водителя решает, когда ему нужно остановиться, пока не будут задействованы тормоза, называется расстоянием мышления (TD). Тормозной путь (BD) — это расстояние, которое проходит автомобиль после нажатия на педаль тормоза до полной остановки. Тормозной путь (SD) — это расстояние мысли плюс тормозной путь, который показан в уравнении 1.

Уравнение 1

Теперь мы можем получить уравнения для TD и BD, используя кинематику и второй закон Ньютона (ΣF = ma).Для этого уравнения потребуется всего несколько переменных: время реакции водителя (th), начальная скорость автомобиля (vo) и коэффициент статического трения между колесами на дороге.

Расстояние мышления (TD)

Давайте сначала начнем с расстояния мышления (TD), которое показано в уравнении 2. Скорость автомобиля можно представить как постоянную в течение короткого промежутка времени, необходимого для реакции водителя, так что все нам нужно это скорость, умноженная на время реакции, чтобы получить расстояние мышления.Поскольку время реакции человека, желающего затормозить, обычно меньше секунды, это расстояние является наименьшим по отношению к тормозному пути.

Уравнение 2

Тормозной путь (BD)

Вывести уравнение для тормозного пути немного сложнее. Начнем с кинематического уравнения, показанного в уравнении 3.

Уравнение 3

Где:

  • vf = конечная скорость
  • vo = начальная скорость
  • a = ускорение
  • d = пройденное расстояние

Мы знаем, что конечная скорость равна нулю, потому что машина остановилась.Единственное, что неизвестно в этом уравнении, — это ускорение a . Автомобиль замедляется (ускоряется в направлении, противоположном его движению), потому что на него действует неуравновешенная сила.

Тормоза создают трение колесам, замедляя их, но статическое трение ( f ) между колесами и дорогой в конечном итоге останавливает машину. Сопротивление воздуха и трение качения участвуют, но в меньшей степени. Вес автомобиля ( мг, ) и нормальная сила ( Н, ) являются вертикальными силами, и они равны.Схема свободного тела показана на Диаграмме 1.

Диаграмма 1.

Второй закон Ньютона используется для расчета ускорения автомобиля. Трение рассчитывается путем умножения коэффициента трения (μ) на нормальную силу ( N ).

f = μ N

Нормальная сила составляет мг , потому что она должна только противодействовать весу автомобиля. Последняя строка в уравнении 4 дает нам ускорение автомобиля.

Уравнение 4

Теперь мы можем подставить ускорение, которое мы только что определили, в уравнение 3, чтобы получить уравнение тормозного пути, BD. Давайте рассмотрим это более подробно.

Уравнение 5. g — ускорение свободного падения.

Последний шаг в нашем выводе уравнения тормозного пути (SD) — это прибавить расстояние мыслей (TD) к тормозному пути (BD), что показано в уравнении 6.

Уравнение 6

Давайте представим, что время реакции нашего водителя составляет 0,5 с, и мы знаем, что начальная скорость составляет 73 км / ч, то есть 20,3 м / с. Коэффициент трения (μ) можно оценить как 0,8, что является средним значением для резиновых шин на сухом бетоне. Что теперь может определить наш минимальный тормозной путь?

Удивительно, что за доли секунды наш мозг может сравнить значение тормозного пути с нашей оценкой того, как далеко мы от перекрестка, и принять решение, остановиться или проехать перекресток.Что ж, может быть, это не совсем то, что происходит, но с практикой вождения мы обучаемся точно оценивать расстояние, которое нам нужно, чтобы остановиться, в зависимости от нашей скорости.

Дальность мышления увеличивается со скоростью. Время нашей реакции может быть постоянным, но умножение его на все более и более высокие скорости увеличивает расстояние мышления с увеличением скорости. Тормозной путь увеличивается экспоненциально с увеличением скорости, потому что начальная скорость автомобиля возводится в квадрат в уравнении тормозного пути.Например, для остановки движения со скоростью 20 м / с требуется дополнительно 24 м по сравнению с 10 м / с. График 1 показывает тормозной путь в сравнении с начальными скоростями.

График 1

Резюме урока

Давайте сделаем несколько минут, чтобы повторить то, что мы узнали!

Каждый раз, когда кто-то водит машину, он должен в какой-то момент остановить ее. Это включает в себя принятие решения об остановке, во время которого автомобиль проезжает определенное расстояние, равное его мгновенной скорости, умноженной на время реакции водителя.Мы называем это расстояние расстоянием мышления (TD). Это кратчайшее расстояние в уравнении тормозного пути, потому что время реакции водителя очень мало.

Тормозной путь (BD) — это расстояние, необходимое для остановки после включения тормозов, а статическое трение между шинами и дорогой является доминирующей тормозящей силой, замедляющей автомобиль до полной остановки.

Сложив эти два расстояния вместе, мы получим тормозной путь (SD).

Самым большим фактором при оценке этого расстояния является скорость автомобиля, поскольку она возводится в квадрат в уравнениях тормозного пути и тормозного пути. {2} \]

Итак, при фиксированной максимальной тормозной силе тормозной путь пропорционален квадрату скорости.

Пример расчета дистанции мышления

Автомобиль движется со скоростью 12 м / с. Водитель имеет время реакции 0,5 с и видит, что впереди на дорогу выбегает кошка. Какова дистанция мышления, когда водитель реагирует?

расстояние = скорость × время

\ [d = v \ times t \]

\ [d = {12} \\ м / с \ times {0,5} \\ s \]

\ [мышление \\ distance = 6 \ m \]

Пример расчета тормозного пути

Автомобиль в предыдущем примере имеет общую массу 900 кг.{2}} {2,000} \]

\ [тормозное \\ расстояние = 32 \\ м \]

Пример расчета тормозного пути

Каков тормозной путь для автомобиля выше?

тормозной путь = расстояние мысли + тормозной путь

тормозной путь = 6 + 32

тормозной путь = 38 м

вопрос

Рассчитайте тормозной путь для автомобиля и водителя в приведенном выше примере, когда движется со скоростью 24 м / с.

Показать ответ

\ [мышление \ расстояние = 24 \ м / с \ умножить на 0.{2}} {100} \]

тормозное усилие ~ 87000 Н

кинематика — Тормозной путь (без трения)

кинематика — Тормозной путь (без трения) — Physics Stack Exchange
Сеть обмена стеков

Сеть Stack Exchange состоит из 176 сообществ вопросов и ответов, включая Stack Overflow, крупнейшее и пользующееся наибольшим доверием онлайн-сообщество, где разработчики могут учиться, делиться своими знаниями и строить свою карьеру.

Посетить Stack Exchange
  1. 0
  2. +0
  3. Авторизоваться Зарегистрироваться

Physics Stack Exchange — это сайт вопросов и ответов для активных исследователей, ученых и студентов-физиков.Регистрация займет всего минуту.

Зарегистрируйтесь, чтобы присоединиться к этому сообществу

Кто угодно может задать вопрос

Кто угодно может ответить

Лучшие ответы голосуются и поднимаются наверх

Спросил

Просмотрено 50к раз

$ \ begingroup $

Предположим, у меня есть тело, путешествующее в космосе со скоростью $ 1000 ~ \ text {m / s} $. 2 $, для остановки потребуется $ 100 с $.2} {2a} $$

Эта формула станет более интересной, когда вы немного изучите физику, потому что это простой пример теоремы работы-энергии.

Добавить комментарий

Ваш адрес email не будет опубликован. Обязательные поля помечены *